0% found this document useful (0 votes)
52 views160 pages

Cadasil

Cadasil neurology

Uploaded by

Satyakiran
Copyright
© © All Rights Reserved
We take content rights seriously. If you suspect this is your content, claim it here.
Available Formats
Download as PDF, TXT or read online on Scribd
0% found this document useful (0 votes)
52 views160 pages

Cadasil

Cadasil neurology

Uploaded by

Satyakiran
Copyright
© © All Rights Reserved
We take content rights seriously. If you suspect this is your content, claim it here.
Available Formats
Download as PDF, TXT or read online on Scribd
You are on page 1/ 160

6/9/24, 12:52 PM Neet PG Preparation, Neet PG Coaching, FMGE, USMLE

Grand Test-22 (Clinical New Pattern)

RESULT

View View View SectionWise


Result
leaderboard Analysis Solution Analysis

Grand Test-22 (Clinical New Pattern)


Question Wise Report

Question: 1
Dorsal pancreatic bud does not form one of the following -

A Head

B Neck

C Body

D Uncinate process

52.46% People got this right

Explanation:

Correct Answer (D) Uncinate process Ref: Read the text below Sol : • The dorsal
bud forms the upper part of head, neck & body of pancreas. • The ventral bud
forms the lower part of head & uncinate process.

Question: 2

https://emedicoz.com/testresult/683510s14692528 1/160
6/9/24, 12:52 PM Neet PG Preparation, Neet PG Coaching, FMGE, USMLE

Which of the following is NOT a branch of the first (mandibular) part of


the first part of the maxillary artery?

A Deep auricular

B Anterior tympanic

C Middle meningeal

D Greater Palatine

25.61% People got this right

Explanation:

Correct Answer (D) Greater Palatine Ref– Read the text below Sol: it is branch
from THIRD pard of artery

Question: 3
Nerve supply of the tip of nose is :

A Infratrochlear

B Supratrochlear

C Supraorbital

D External nasal

35.17% People got this right

Explanation:

https://emedicoz.com/testresult/683510s14692528 2/160
6/9/24, 12:52 PM Neet PG Preparation, Neet PG Coaching, FMGE, USMLE

Correct Answer (D) External nasal Ref: Read the text below Sol : The tip of nose is
supplied by external nasal a branch of ophthalmic division of trigeminal nerve.

Question: 4
Structures passing through lesser sciatic foramen are all except:

A Superior gluteal nerve

B Obturator internus tendon

C Internal pudendal vessels

D Pudendal nerve

41.51% People got this right

Explanation:

Correct Answer (A) Superior gluteal nerve Ref: Read the text below Sol : • The
structures passing into the lesser sciatic foramen & thus into the perineum are
Pudendal nerve, Internal pudendal vessels & nerve to obturator internus.( PIN ) •
The structure emerging through the foramen is the tendon of obturator
internus. The superior gluteal nerve leaves through the greater sciatic foramen.

Question: 5
While removing a fish bone stuck in the throat, the main trunk of internal
laryngeal nerve was severed. The bone was stuck in the :

A Piriform fossa

B Sinus of larynx

https://emedicoz.com/testresult/683510s14692528 3/160
6/9/24, 12:52 PM Neet PG Preparation, Neet PG Coaching, FMGE, USMLE

C Vallecula epiglottis

D Vestibule of larynx

51.15% People got this right

Explanation:

Correct Answer (A) Piriform fossa Ref: Read the text below Sol : On either side of
the laryngeal orifice is a recess, termed the piriform sinus (also piriform recess,
pyriform sinus, piriform fossa, or smuggler's fossa), which is bounded medially
by the aryepiglottic fold, laterally by the thyroid cartilage and thyrohyoid
membrane. The fossae are involved in speech. The term "piriform," which means
"pear-shaped," is also sometimes spelled &"pyriform" (as in the diagram on this
page.) Deep to the mucous membrane of the piriform fossa lie the recurrent
laryngeal nerve as well as the internal laryngeal nerve, a branch of the superior
laryngeal nerve. The internal laryngeal nerve supplies sensation to the area, and
it may become damaged if the mucous membrane is inadvertently punctured

Question: 6
Which of the following landmark is not used for the Identification of
facial nerve during parotidectomy?

A Stylomastoid foramen,

B Posterior belly of digastric


(PBD),

C Tragal pointer (TP),

D Omohyoid

49.95% People got this right

Explanation:

https://emedicoz.com/testresult/683510s14692528 4/160
6/9/24, 12:52 PM Neet PG Preparation, Neet PG Coaching, FMGE, USMLE

Correct Answer (D)

Conventional antegrade dissection of the facial nerve, and retrograde dissection.


Numerous soft tissue and bony landmarks have been proposed to assist the
surgeon in the early identification of this nerve. Most commonly used
anatomical landmarks to identify facial nerve trunk are stylomastoid foramen,
tympanomastoid suture (TMS), posterior belly of digastric (PBD), tragal pointer
(TP), mastoid process and peripheral branches of the facial nerve.

Question: 7
Stafne bone cyst is a?

A Benign bone cyst

B Degenerative change in
cortical part of bone

C Aneurysmal bone cyst

D Ectopic salivary gland

24.99% People got this right

Explanation:

Correct Answer (D)

https://emedicoz.com/testresult/683510s14692528 5/160
6/9/24, 12:52 PM Neet PG Preparation, Neet PG Coaching, FMGE, USMLE

Stafne bone cyst or static bone cavity of the mandible, is salivary gland inclusion
defect near the angle of the mandible below the mandibular canal. It is usually
an incidental finding and represents a depression in the medial aspect of the
mandible filled by part of the submandibular gland or adjacent fat.

Question: 8
Hartley Dunhill operation is?

A A type of
hemithyroidectomy done for
Toxic nodule

B Subtotal thyroidectomy
done for toxic nodular goitre

C Total thyroidectomy done


for diffuse toxic goitre

D One side lobectomy with


isthmectomy contra-lateral
subtotal thyroidectomy done for
non toxic multinodular goitre.

39.51% People got this right

Explanation:

https://emedicoz.com/testresult/683510s14692528 6/160
6/9/24, 12:52 PM Neet PG Preparation, Neet PG Coaching, FMGE, USMLE

Correct Answer (D)

Question: 9
What is the grade of frostbite in the Image shown below?

A Grade I

B Grade II

C Grade III

D Grade IV

34.11% People got this right

Explanation:

https://emedicoz.com/testresult/683510s14692528 7/160
6/9/24, 12:52 PM Neet PG Preparation, Neet PG Coaching, FMGE, USMLE

Correct Answer (C) (Haemorrhagic blisters) Frostbite staging: • First degree -


numbness, central pallor, surrounding erythema/edema, desquamation,
dysesthesia • Second degree - skin blistering with surrounding erythema/edema
• Third degree - tissue loss involving entire thickness of skin, hemorrhagic
blisters • Fourth degree - tissue loss involving deeper structures, resulting in loss
of the affected part Treatment- Remove patients from the wind. Remove wet
clothing and replace with dry clothing. Avoid vigorous rubbing as this can cause
further damage. Warm water baths, approximately 40-42 degrees C. Warm IV
fluids, and this should precede warming of the affected extremity. NSAIDS
(ibuprofen). White, cloudy-appearing blisters can be drainedbut hemorrhagic
blisters should be left intact. Prevent infections and dehydration. Complete
rewarming should be achieved before surgical debridement to demarcate viable
tissue. Patients with full-thickness injuries and evidence of ischemia and no
restoration of tissue perfusion after rewarming may be candidates for
thrombolytic (tPA) therapy.

Question: 10
Which of the following is false about parathyroid?

A Parathyroids are located


near junction of RLN and inferior
thyroid artery

B Superior parathyroid take


supply from superior thyroid
artery and inferior parathyroid
from inferior thyroid artery.

C Superior parathyroid are


deep while Inferior parathyroid
are superficial to RLN

D Most common location of


ectopic parathyroid is
paraesophageal.

30.26% People got this right

https://emedicoz.com/testresult/683510s14692528 8/160
6/9/24, 12:52 PM Neet PG Preparation, Neet PG Coaching, FMGE, USMLE

Explanation:

Correct Answer (B) All 4 parathyroid take supply from inferior thyroid artery

Question: 11
Treatment of choice for stage pTa bladder cancer is?

A Cystoscopic tumor
resection

B Cystoscopic tumor
resection and intravesical BCG

C Cystoscopic tumor
resection and intravesical
chemotherapy

D Partial cystectomy and


intravesical chemotherapy.

8.93% People got this right

Explanation:

Correct Answer (A) Cystoscopic tumor resection

Question: 12
Which of the following conditions give Spiderweb (or cobweb sign)? 1:
Corona infection (Lung) 2: Dissecting aneurism of aorta 3: Budd Chiari
syndrome 4: Peri-renal edema

A 1

https://emedicoz.com/testresult/683510s14692528 9/160
6/9/24, 12:52 PM Neet PG Preparation, Neet PG Coaching, FMGE, USMLE

B 1&2

C 1, 2 & 3

D 1, 2, 3 & 4

21.11% People got this right

Explanation:

Correct Answer (D)

Question: 13
Most common complication associated with prolonged Lloyd Davis
position is?

A Calf compartment
syndrome

B Peroneal nerve injury

C Sciatic nerve compression

D CHF due to increased

https://emedicoz.com/testresult/683510s14692528 10/160
6/9/24, 12:52 PM Neet PG Preparation, Neet PG Coaching, FMGE, USMLE

venous return

12.78% People got this right

Explanation:

Correct Answer (A)

The Lloyd Davies position is used to gain access to the pelvis for gynaecological,
urological and colorectal procedures. Prolonged position (> 4 h) has been
associated with the development of bilateral compartment syndrome of the
calves.

Question: 14
The procedure shown in the image is done for?

A Grade I and II
haemorrhoids

https://emedicoz.com/testresult/683510s14692528 11/160
6/9/24, 12:52 PM Neet PG Preparation, Neet PG Coaching, FMGE, USMLE

B 2nd and some 3rd degree


haemorrhoids

C 3rd and 4th degree


haemorrhoids

D Only 4th degree


haemorrhoids

26.73% People got this right

Explanation:

Correct Answer (B) 2nd and some 3rd degree haemorrhoids

Question: 15
For the condition shown in the “picture A” a procedure shown in “picture B”
was done from perineal approach. Which of the following procedure has
been done in this patient?

A Delorme operation

B Frykman Goldberg
operation

https://emedicoz.com/testresult/683510s14692528 12/160
6/9/24, 12:52 PM Neet PG Preparation, Neet PG Coaching, FMGE, USMLE

C Ripstein operation

D Altemeier operation

13.33% People got this right

Explanation:

Correct Answer (D) Altemeier is perineal recto-sigmoidectomy for rectal


prolapse.

Question: 16
False statement about sympathectomy is?

A Best results are seen in


cases of Hyperhydrosis

B Excision of L1 may led to


impotency or dry ejaculation

C In Burger’s disease
Intermittent claudication
responds well to sympathectomy

D During surgery it may be


confused with Psoas minor or
genitofemoral nerve

24.33% People got this right

Explanation:

Correct Answer (C) In Burger’s disease Intermittent claudication responds well to


sympathectomy

https://emedicoz.com/testresult/683510s14692528 13/160
6/9/24, 12:52 PM Neet PG Preparation, Neet PG Coaching, FMGE, USMLE

Question: 17
Intratubular Germ cell tumor is found in?

A Spermatocytic seminoma

B Embryonal cell cancer

C Paediatric yolk sac tumour

D Paediatric teratoma

14.17% People got this right

Explanation:

Correct Answer (B) Intratubular germ cell neoplasia (ITGCN) is the precursor
lesion for invasive testicular germ cell tumors (TGCTs) of adolescents and young
adults. While initially termed carcinoma in situ (CIS) or testicular intraepithelial
neoplasia (TIN), now intratubular germ cell neoplasia (ITGCN) is prefereed. It has
a high risk of progression to testicular cancer. It is never found with
Spermatocytic seminoma, Pediatric yolk sac or benign teratoma. Treated by
orchidectomy (TOC), surveillance or Radiotherapy. Chemotherapy has high rate
of recurrence.

Question: 18
Most common cause of Post TURP delayed obstructive symptoms?

A Stricture at Fossa
navicularis

B Stricture at membranous
urethra

https://emedicoz.com/testresult/683510s14692528 14/160
6/9/24, 12:52 PM Neet PG Preparation, Neet PG Coaching, FMGE, USMLE

C Stricture at Bulb of urthra

D Bladder neck contracture

23.98% People got this right

Explanation:

Correct Answer (D) Bladder neck contracture after TURP is seen when extensive
TURP is done in a small size prostate (< 25 grams). Treated by bladder neck
incision using Collins knife making two incisions at 5 and 7 o clock positions

Question: 19
In the treatment of prostate cancer which drug does not need anti-
androgens to preven LH surge and tumor flare?

A Luprolide

B Goserelin

C Triptorelin

D Degarelix

28.43% People got this right

Explanation:

Correct Answer (D)

Luprolide, Goserelin, Triptorelin are LHRH agonist while Degarelix is LHRH


antagonist.

https://emedicoz.com/testresult/683510s14692528 15/160
6/9/24, 12:52 PM Neet PG Preparation, Neet PG Coaching, FMGE, USMLE

Question: 20
Which of the following is not a sign of G.I. TB?

A Fleischner Sign

B Sterlin sign

C Chicken intestine

D Frostberg sign

22.04% People got this right

Explanation:

Correct Answer (D)

https://emedicoz.com/testresult/683510s14692528 16/160
6/9/24, 12:52 PM Neet PG Preparation, Neet PG Coaching, FMGE, USMLE

Question: 21
Not included in Damage control surgery?

A Decontamination

B Hemorrhage control

C Resuscitation

D Vascular anastomosis

45.82% People got this right

Explanation:

Correct Answer (D)

https://emedicoz.com/testresult/683510s14692528 17/160
6/9/24, 12:52 PM Neet PG Preparation, Neet PG Coaching, FMGE, USMLE

Question: 22
A 40 year lady presented with a small swelling in lower abdomen which
she noticed 2 years ago. On examination a transverse scar was present in
suprapubic area with a non-tender swelling protruding through lateral
area of the scar. Cough impulse was positive. She gave a history of
caesarean section at the age of 25 years. The scar is 4 cm above the pubic
symphysis. According to EHS hernia classification this hernia is which type
of hernia?

A M2

B M3

C M4

D M5

19.83% People got this right

Explanation:

Correct Answer (C)

https://emedicoz.com/testresult/683510s14692528 18/160
6/9/24, 12:52 PM Neet PG Preparation, Neet PG Coaching, FMGE, USMLE

Question: 23
A doctor was called in emergency for a RTA patient. He emergency is saw
a 32 year old male patient who is dyspnoeic. On examination his pulse is
100/min and BP is 106/72 mmHg. Abdominal examination is normal but
left sided chest showed tenderness over 4 ribs with paradoxical
movement. Patient Pa02 is 60mmHg. In the remote area where he is
posted he does not have facility of positive pressure ventilation. What is
the best treatment for this patient?

A Strapping

B O2 administration

C Intrapleural local analgesia

D Towel clip traction or


external fixation of flail segment

25.2% People got this right

Explanation:

Correct Answer (D) Towel clip traction or external fixation of flail segment

https://emedicoz.com/testresult/683510s14692528 19/160
6/9/24, 12:52 PM Neet PG Preparation, Neet PG Coaching, FMGE, USMLE

Question: 24
Regarding breast cancer match the following according to their property?

A A-4, B-3, C-2, D-1

B A-4, B-1, C-2, D-3

C A-3, B-4, C-2, D-1

D A-2, B-3, C-1, D-4

36.43% People got this right

Explanation:

Correct Answer (A) A-4, B-3, C-2, D-1

Question: 25
False statement about papillary thyroid cancer?

A Multicentric

B Psammoma bodies

https://emedicoz.com/testresult/683510s14692528 20/160
6/9/24, 12:52 PM Neet PG Preparation, Neet PG Coaching, FMGE, USMLE

C Node spread common

D Hurthle cell is a subtype

41.34% People got this right

Explanation:

Correct Answer (D) Hurthle cell is a subtype

Question: 26
False statement about neuroblastoma?

A Karyorrhexis and nuclear


pleomorphism common

B Lung metastasis common

C Homer wright rosette

D Raccoon eye and blue


berry muffins seen

21.65% People got this right

Explanation:

Correct Answer (B)

Commonest site of metastasis is bone (Skin and liver in 1s)

https://emedicoz.com/testresult/683510s14692528 21/160
6/9/24, 12:52 PM Neet PG Preparation, Neet PG Coaching, FMGE, USMLE

Question: 27
Drooping lily sign is found in?

A Neuroblastoma and renal


duplication

B Renal duplication and


hydatid disease

C Neuroblastoma and
hydatid disease

D Hydatid disease and Wilm’s


tumor

18.41% People got this right

Explanation:

Correct Answer (A)

https://emedicoz.com/testresult/683510s14692528 22/160
6/9/24, 12:52 PM Neet PG Preparation, Neet PG Coaching, FMGE, USMLE

Question: 28
A patient presented with a swelling in thyroid. The radionuclide scan
showed a cold nodule and USG showed a 3 cm non cystic solid mass.
FNAC did not show any malignant cell. What is the best treatment for this
patient?

A Lobectomy

B Antithyroid drugs

C Hemithyroidectomy

D Subtotal thyroidectomy

25.1% People got this right

Explanation:

Correct Answer (C) Hemithyroidectomy

Question: 29
False statement about Pendred syndrome is?

https://emedicoz.com/testresult/683510s14692528 23/160
6/9/24, 12:52 PM Neet PG Preparation, Neet PG Coaching, FMGE, USMLE

A It is a autosomal dominant
condition

B Defect is in Chromosome
7q

C Presents with hypothyroid


and sensorineural hearing loss

D Enlarged vestibular
aqueduct

23.7% People got this right

Explanation:

Correct Answer (A) Pendred syndrome is Autosomal recessive condition

Question: 30
Identify the suturing technique?

A Vertical mattress

B Horizontal mattress

https://emedicoz.com/testresult/683510s14692528 24/160
6/9/24, 12:52 PM Neet PG Preparation, Neet PG Coaching, FMGE, USMLE

C Tension sutures

D Interrupted skin suturing

39.05% People got this right

Explanation:

Correct Answer (B) Horizontal mattress

Question: 31
Which of the following is true about fracture shown in children?

A Anterior displacement of
the distal fragment is more
common than posterior

B Cubitus valgus is more


common than cubitus varus
during mal-union

C The neurological
complications are transitory

https://emedicoz.com/testresult/683510s14692528 25/160
6/9/24, 12:52 PM Neet PG Preparation, Neet PG Coaching, FMGE, USMLE

D Weakness of elbow flexion


is a common complication

14.64% People got this right

Explanation:

Correct Answer (C) The neurological complications are transitory Ref: Apley’s
‘Textbook of Orthopaedics and Fractures’; 9/e, Chap 24 Sol: SUPRACONDYLAR
FRACTURES • These are among the commonest fractures in children. • The
distal fragment may be displaced either posteriorly or anteriorly. MECHANISM
OF INJURY: • Posterior angulation or displacement (95 per cent of all cases)
suggests a hyperextension injury, usually due to a fall on the outstretched hand.
• The humerus breaks just above the condyles. • The distal fragment is
pushed backwards and (because the forearm is usually in pronation) twisted
inwards. • The jagged end of the proximal fragment pokes into the soft tissues
anteriorly sometimes injuring the brachial artery or median nerve. • Anterior
displacement is rare (option ‘1’); it is thought to be due to direct violence (e.g. a
fall on the point of the elbow) with the joint in flexion. CLINICAL FEATURES: •
Following a fall, the child is in pain and the elbow is swollen; with a posteriorly
displaced fracture the S-deformity of the elbow is usually obvious and the bony
landmarks are abnormal. • It is essential to feel the pulse and check the
capillary return; passive extension • of the flexor muscles should be pain-free.
• The wrist and the hand should be examined for evidence of nerve injury. •
The fracture is seen most clearly in the lateral view. • In an undisplaced
fracture the ‘fat pad sign’ should raise suspicions: there is a triangular lucency in
front of the distal humerus, due to the fat pad being pushed forwards by a
haematoma. • The radial nerve, median nerve (particularly the anterior
interosseous branch) or the ulnar nerve may be injured. • Fortunately loss of
function is usually temporary and recovery can be expected in 3 to 4 months
(option ‘3’). • If there is no recovery the nerve should be explored. • However,
if a nerve, documented as intact prior to manipulation, is then found to have
failed after manipulation, then entrapment in the fracture is suspected and
immediate exploration should be arranged. COMPLICATIONS: • Malunion is
common. • However, backward or sideways shifts are gradually smoothed out
by modelling during growth and they seldom give rise to visible deformity of the
elbow. • Forward or backward tilt may limit flexion or extension, but
consequent disability is slight. • Weakness of elbow extension is commonly
observed (option ‘4’). • Uncorrected sideways tilt (angulation) and rotation are
much more important and may lead to varus (or rarely valgus) deformity of the
elbow (option ‘2’); this is permanent and will not improve with growth (gun-
stock deformity). • The fracture is extra-physeal and so physeal damage should
not be blamed for the deformity; usually it is faulty reduction which is
https://emedicoz.com/testresult/683510s14692528 26/160
6/9/24, 12:52 PM Neet PG Preparation, Neet PG Coaching, FMGE, USMLE

responsible. • Cubitus varus is disfiguring and cubitus valgus may cause late
ulnar palsy. • If deformity is marked, it will need correction by supracondylar
osteotomy usually once the child approaches skeletal maturity.

Question: 32
Maxpage Surgery is :

A Distal Sliding of Common


Flexor Origin

B Proximal Sliding of
Common Flexor Origin

C Distal Sliding of Common


Extensor Origin

D Proximal Sliding of
Common Extensor Origin

26.24% People got this right

Explanation:

Correct Answer (A) Distal Sliding of Common Flexor Origin

Question: 33
A 42 year old female presents with tingling and numbness in right hand
that awakens her in the middle of the night for last two weeks. She also
gives history of being treated for diabetes and hypothyroidism for last 3
years by an endocrinologist. Which of the following ain’t true about this
condition from the following options ?

A Splintage is helpful only in


few initial months of onset of

https://emedicoz.com/testresult/683510s14692528 27/160
6/9/24, 12:52 PM Neet PG Preparation, Neet PG Coaching, FMGE, USMLE

symptoms

B Motor Symptoms appear


early in the course of the disease

C Durakn’s Test is the most


specific clinical test in the
diagnosis

D It is the most common


focal compression neuropathy

42.55% People got this right

Explanation:

Correct Answer (B) Motor symptoms are the last to appear

Question: 34
Identify the fracture below :

https://emedicoz.com/testresult/683510s14692528 28/160
6/9/24, 12:52 PM Neet PG Preparation, Neet PG Coaching, FMGE, USMLE

A Barton’s fracture

B Chaueffer’s fracture

C Smith’s fracture

D Colles’ fracture

32.58% People got this right

Explanation:

Correct Answer (B) Intra articular fracture of distal end radius with radial styloid
fragment with intact radiocarpal joint anatomy : Chaueffer’s / Hutchinson /
Backfire Fracture

Question: 35
Steps in the operative correction of congenital talipes equinovarus
include all of the following except

A Lengthening of
tendoachilles

B Posterior tibiotalar
capsulotomy

C Release of peroneus brevis


from its insertion

D Talonavicular joint
reduction

26.46% People got this right

https://emedicoz.com/testresult/683510s14692528 29/160
6/9/24, 12:52 PM Neet PG Preparation, Neet PG Coaching, FMGE, USMLE

Explanation:

Correct Answer (C) Release of peroneus brevis from its insertion The crucial
component in CTEV is subluxation of the talonavicular joint. The soft tissues on
the medial side of the foot are underdeveloped and shorter than normal. Aim of
treatment is to set the tarsal bones in normal relationship to one another and to
relieve deforming stresses thus allowing the bones to develop in their normal
shape from an early age. All taut ligaments at the medial side of the ankle and
foot are divided and any tendon that is too tight to allow full correction is
lengthened including the calcaneal tendon (tendoAchilles). Finally the tarsal
bones thus released are restored to their normal relationships, particular
attention being paid to the talus and navicular bone. The peroneus brevis is
present on the lateral side of the foot and is an evertor; hence it is not released.

Question: 36
In patients with hypertrophic cardiomyopathy maximum mutations are
found in which gene:

A b - myosin heavy chain

B Elastin

C a – tropomyosin

D Troponin T

48.44% People got this right

Explanation:

Correct Answer (A) b - myosin heavy chain Mutations in gene for b - Mysoin
heavy chain are associated with 40% of the families with hypertrophic
cardiomyopathy. • Troponin T mutations - 15% of the families • α - tropomyosin
mutations ~5% of the families

https://emedicoz.com/testresult/683510s14692528 30/160
6/9/24, 12:52 PM Neet PG Preparation, Neet PG Coaching, FMGE, USMLE

Question: 37
Giant "a" waves in the jugular vein can occur all of following except:

A Constrictive pericarditis.

B Pulmonary hypertension.

C Aortic regurgitation.

D Tricuspid stenosis.

43.04% People got this right

Explanation:

Correct Answer (C) Aortic regurgitation. • "a" waves are the positive deflection in
the jugular venous pulse following right atrial contraction. • This becomes giant
in situations where the atrium is hypertrophied or contracts against resistance,
such as constrictive pericarditis, pulmonary hypertension, and tricuspid stenosis
or atresia

Question: 38
Drug used in ventricular arrhythmia due to WPW syndrome is :

A Amiodarone

B Digoxin

C Lignocaine

D Verapamil

34% People got this right


https://emedicoz.com/testresult/683510s14692528 31/160
6/9/24, 12:52 PM Neet PG Preparation, Neet PG Coaching, FMGE, USMLE

Explanation:

Correct Answer (A)

Amiodarone

Amiodarone has been found effective in a wide range of ventricular and


supraventricular arrhythmias.
Resistant VT and recurrent VF are the most important indications. It is also used
to maintain sinus rhythm in AF when other drugs have failed.
Rapid termination of ventricular and supraventricular arrhythmias can be
obtained by I.V. injection.
WPW tachyarrhythmia is terminated by suppression of both normal and
aberrant pathways.

Question: 39
Patient comes to ER with chest pain of 3 hours duration and normal BP.
ECG shows ST depression with ‘t’ inversion. Troponin levels are 140. Next
line of management?

A PCI

B Thrombolysis with
Alteplase

C Prophylaxis for arrythmia

D Aspirin with Heparin

27.39% People got this right

Explanation:

Correct Answer (D) NSTEMI – Hence Aspirin with Heparin

Question: 40
https://emedicoz.com/testresult/683510s14692528 32/160
6/9/24, 12:52 PM Neet PG Preparation, Neet PG Coaching, FMGE, USMLE

Chest pain with ST elevation on ECG. Best test for diagnosis within 2-4
hours

A Troponin T

B BNP

C CpkMB

D FABP

11.14% People got this right

Explanation:

Correct Answer (D) Heart specific Fatty acid binding protein rises in 2 hours.
Troponin and CpkMB rises 6-8 and 4-6 hours later respectively. BNP is a marker
of Heart failure and not used for diagnosis of ACS.

Question: 41
The presence of which of the following conditions portends the poorest
longterm prognosis in patients with aortic stenosis?

A Angina

B Syncope

C Valve area of less than


0.5cm2

D CHF

18.51% People got this right

https://emedicoz.com/testresult/683510s14692528 33/160
6/9/24, 12:52 PM Neet PG Preparation, Neet PG Coaching, FMGE, USMLE

Explanation:

Correct Answer (D) CHF • CHF has the poorest long-term prognosis at 2 years,
while syncope and angina have a prognosis of 3 and 5 years respectively. The
valve area or calcification offer no information concerning prognosis

Question: 42
A 26-year-old woman with prior history of IV drug abuse resulting in
hepatitis C coinfection is seen in a prenatal clinic for routine care. She is
in the third trimester of pregnancy with her first child. Which of the
following statements regarding transmission of hepatitis C virus (HCV) to
neonate is true?

A Caesarean section reduces


the risk of transmission to a
greater degree than vaginal
delivery.

B Risk of transmission is 5%
to neonate.

C Breastfeeding is associated
with very high risk of
transmission.

D Patients with HCV


genotype 1 have a greater risk of
transmission than other
genotypes.

20.4% People got this right

Explanation:

Correct Answer (B) The vertical risk of transmission is 5%. Breastfeeding is not
associated with high risk of transmission. The mode of delivery (vaginal versus

https://emedicoz.com/testresult/683510s14692528 34/160
6/9/24, 12:52 PM Neet PG Preparation, Neet PG Coaching, FMGE, USMLE

caesarean section), and viral genotype are not associated with increased risk of
vertical transmission of hepatitis C virus.

Question: 43
Which of the following are components of MELD’s score?

A Serum creatinine, total


bilirubin, Prothrombin time

B Serum creatinine, total


bilirubin, INR

C Serum AST, total bilirubin,


Prothrombin time

D Serum ALT, total bilirubin,


INR

42.95% People got this right

Explanation:

Correct Answer (B) MELD’s score MELD = 9.57 × log(creatinine)+ 3.78 × log
(total bilirubin)+11.2 × log (INR) + 6.43

Question: 44
Identify the test shown in the figure

https://emedicoz.com/testresult/683510s14692528 35/160
6/9/24, 12:52 PM Neet PG Preparation, Neet PG Coaching, FMGE, USMLE

A Fluid thrill test

B Shifting dullness

C Puddle sign

D Horseshoe shape dullness


percussion

66.14% People got this right

Explanation:

Correct Answer (A) Fluid thrill test Positive Fluid thrill indicates tense ascites. No
relation to amount of fluid.

Question: 45
Drug of choice for spontaneous bacterial peritonitis?

A Ceftriaxone

B Amoxicillin

C Cefotaxime

https://emedicoz.com/testresult/683510s14692528 36/160
6/9/24, 12:52 PM Neet PG Preparation, Neet PG Coaching, FMGE, USMLE

D Metronidazole

24.19% People got this right

Explanation:

Correct Answer (C) Cefotaxime

Question: 46
Orthodeoxia is

A Dyspnoea increases in
sitting position and decreases in
supine position

B Dyspnoea increases in
supine position and decreases in
sitting position

C Oxygen saturation
increases in sitting position and
decreases in supine position

D Oxygen saturation
increases in supine position and
decreases in sitting position

14.45% People got this right

Explanation:

Correct Answer (A) Platypnoea- Dyspnoea increases in sitting position and


decreases in supine position.

Question: 47
https://emedicoz.com/testresult/683510s14692528 37/160
6/9/24, 12:52 PM Neet PG Preparation, Neet PG Coaching, FMGE, USMLE

Which of the following is false regarding diagnostic criteria of hepato-


renal syndrome?

A Serum creatinine level ≥1.5


mg/dl

B Urinary RBCs < 50 HPF

C Urine Protein > 500


mg/day

D Presence of ascites

17.8% People got this right

Explanation:

Correct Answer (C)

Urine protein > 500mg/day

Question: 48
A patient with H. Pylori infection is treated with drugs. The preferred test
to detect the presence of residual H. Pylori infection in this person is-

https://emedicoz.com/testresult/683510s14692528 38/160
6/9/24, 12:52 PM Neet PG Preparation, Neet PG Coaching, FMGE, USMLE

A Rapid urease test

B Urea breath test

C Endoscopy and biopsy

D Serum anti H. Pylori titre

48.66% People got this right

Explanation:

Correct Answer (B) As it is non-invasive test.

Question: 49
A Patient presents with prognathism, enlarged hand & feet. Most likely
transcription factor involved in the disorder is: -

A SF-1.

B DAX -1.

C GATA -2.

D Pit-1.

22.36% People got this right

Explanation:

Correct Answer (D) Pit -1. Prognathism with enlarged hand & feet is suggestive
of ACROMEGALY. M.C.C of Acromegaly is SOMATOTROPHIC ADENOMA.

https://emedicoz.com/testresult/683510s14692528 39/160
6/9/24, 12:52 PM Neet PG Preparation, Neet PG Coaching, FMGE, USMLE

Somatotrophs, Lactotrophs & Thyrotropes have a common transcription factor


Pit -1, Prop -1 While Gonadotrophs have SF-1 & DAX 1 as its transcription factor.

Question: 50
A 45-year-old female presents with weight gain & lethargy on
investigations her TSH levels are high & T4 levels are low. Likely diagnosis
for her clinical condition is: -

A Graves’ disease.

B Hashimoto disease.

C Pituitary dysfunction.

D Hypoparathyroidism.

53.85% People got this right

Explanation:

Correct Answer (B) Hashimoto disease. Lethargy with weight gain is


symptomatically consistent with Hypothyroidism. Lab investigations revealing
high TSH & low T4 is suggestive of Hashimoto disease. In Graves disease
symptoms of Hyperthyroidism are seen with low TSH & high T4. In pituitary
dysfunction (hypofunctional) can cause secondary hypothyroidism but in it TSH
& T4 both are low.

Question: 51
Causes of secondary hyper parathyroidism are all EXECEPT?

A Vitamin D deficiency.

https://emedicoz.com/testresult/683510s14692528 40/160
6/9/24, 12:52 PM Neet PG Preparation, Neet PG Coaching, FMGE, USMLE

B GI malabsorption.

C Chronic kidney disease.

D Parathyroid adenoma.

55.22% People got this right

Explanation:

Correct Answer (D) Parathyroid adenoma. Secondary hyper parathyroidism


occurs in the setting of hypocalcaemia to elevate the calcium levels. Common
conditions causing secondary hyper parathyroidism are:- 1). Chronic kidney
disease. 2). Vitamin D deficiency. 3). Malabsorption. Primary parathyroidism
occurs most commonly due to PARATHYROID ADENOMA. While tertiary hyper
parathyroidism is parathyroid ADENOMA arising from parathyroid
HYPERPLASIA.

Question: 52
Which of the following statements are true of primary
hyperparathyroidism?

A It is associated with
hypocalciuria due to elevated PTH
levels.

B PTH is secreted in a
pulsatile manner from the
posterior pituitary and acts
through PTH receptors on
parathyroid cell membranes

C It is usually caused by an
adenoma of a single parathyroid
gland.

https://emedicoz.com/testresult/683510s14692528 41/160
6/9/24, 12:52 PM Neet PG Preparation, Neet PG Coaching, FMGE, USMLE

D It progresses to tertiary
hyperparathyroidism with time.

39.1% People got this right

Explanation:

Correct Answer (C) It is usually caused by an adenoma of a single parathyroid


gland. • "Primary HPT can be divided pathologically into adenoma, hyperplasia,
and carcinoma. Adenomas clearly are the most prevalent entity representing 80-
85% of cases. • Hyperplasia is the second most common diagnosis constituting
15% of cases. Carcinoma represents <1% of total cases. • Double adenoma has
been found in approximately 5% of the time, and complicates the clinical
distinction between adenoma and hyperplasia. • Histologically, normal
parathyroid tissue shows a cell to fat ratio of 1:1. Hypercellular parathyroid tissue
is typified by the loss of the normal amount of fat. • In primary
hyperparathryroidism there is usually hypercalciuria. Secondary
hyperparathyroidism may progress to tertiary but primary does not.

Question: 53
A Dengue patient, 24 hours without fever, no nausea/vomiting. Had
epistaxis which resolved spontaneously. Hemodynamically stable. Lab is
normal except platelet count of 14000. Which is correct statement
regarding further management?

A 4 units of platelet

B IVFLUIDS

C Oral hydration

D Observation

16.66% People got this right

Explanation:

https://emedicoz.com/testresult/683510s14692528 42/160
6/9/24, 12:52 PM Neet PG Preparation, Neet PG Coaching, FMGE, USMLE

Correct Answer (C) Oral hydration

Question: 54
Anaemia with hyperpigmented knuckles

A Anaemia with
hyperpigmented knuckles

B Thiamine deficiency

C Iron deficiency

D Wilsons disease

29.25% People got this right

Explanation:

Correct Answer (A) B12 deficiency Hyperpigmented knuckles are seen in around
19% of Megaloblastic anaemia cases. However, the finding when present is
specific for B12 deficiency

Question: 55
A 26-year-old man is found to have acute leukemia. Three weeks after
initiation of chemotherapy he has a body temperature spike to 38.9 C
(102 F), with a white blood cell count of 500/mm. Which of the following
is true regarding this clinical situation?

A Antibiotics should be given


if a source of infection can be
identified

https://emedicoz.com/testresult/683510s14692528 43/160
6/9/24, 12:52 PM Neet PG Preparation, Neet PG Coaching, FMGE, USMLE

B Normal chest
roentgenographic findings would
eliminate pneumonia as the cause
of the fever

C Bacteremia, if present,
would most likely be caused by
pseudomonas and other gram-
negative bacilli

D Chemotherapy and tumor


lysis are the most likely cause of
fever

9.48% People got this right

Explanation:

Correct Answer (C) Bacteremia, if present, would most likely be caused by


pseudomonas and other gram-negative bacilli • The leading cause of death in
cancer patients is infection. Fever developing during the course of treatment for
cancer demands careful investigation for infection by all classes of pathogens:
bacteria, viruses, yeasts, and protozoans. • Pneumonia is common but difficult to
diagnose. There is scant sputum production, and often the chest
roentgenographic findings are misleadingly normal. Gram-negative bacilli are
responsible for most pneumonias and bacteremias. • The skin, mucous
membranes, and catheter sites are common portals of entry. In the absence of
an obvious source of infection, it is essential to begin empirical antibiotic
therapy aimed at the most common organisms. • An antipseudomonal penicillin
plus an aminoglycoside is a commonly used regimen. • Both chemotherapy and
the tumor lysis syndrome can cause fever, but infection is much more likely to
be the cause

Question: 56
A 60-year-old man with malignant lymphoma presents with the findings
seen in the picture given below. These findings last for 2 to 4 days and recur
periodically. This condition differs from its related hereditary form by low
levels of

https://emedicoz.com/testresult/683510s14692528 44/160
6/9/24, 12:52 PM Neet PG Preparation, Neet PG Coaching, FMGE, USMLE

A C1 (INH)

B C1

C C4

D C5

19.99% People got this right

Explanation:

Correct Answer (A) C1INH • The picture shows an angioedematous eruption on


the right hand, which is associated with the absence of C1 (INH). •
Lymphoproliferative disorders such as malignant lymphoma are associated with
an acquired form of C1 (INH) deficiency that differs from hereditary angioedema
(familial C1 (INH) deficiency) in its lack of a familial element and reduction in
C1/HB. C2 and C4 are reduced in both disorders. • C5a is an anaphylatoxin.

Question: 57
Inherited causes of Aplastic anaemia are all except:

A Fanconi’s anaemia

https://emedicoz.com/testresult/683510s14692528 45/160
6/9/24, 12:52 PM Neet PG Preparation, Neet PG Coaching, FMGE, USMLE

B Schwachman-Diamond
syndrome

C Preleukemia

D Paroxysmal nocturnal
haemoglobinuria

36.78% People got this right

Explanation:

Correct Answer (D) PNH is an acquired defect leading to intra-vascular


haemolysis. Eventually patients develop Aplastic anaemia.

Question: 58
Tumor lysis syndrome (TLS) is characterized by all except:

A Hyperuricemia

B Hyperkalemia,

C Hyperphosphatemia

D Hypercalcemia

39.3% People got this right

Explanation:

Correct Answer (D) Hypercalcemia Tumor lysis syndrome (TLS) is an oncologic


emergency that is caused by massive tumor cell lysis with the release of large
amounts of potassium, phosphate, and nucleic acids into the systemic
circulation. The metabolic consequences include hyperkalemia,
hyperphosphatemia, secondary hypocalcemia, hyperuricemia, and acute kidney
injury. High levels of both uric acid and phosphate increase the severity of acute
https://emedicoz.com/testresult/683510s14692528 46/160
6/9/24, 12:52 PM Neet PG Preparation, Neet PG Coaching, FMGE, USMLE

kidney injury because uric acid precipitates readily in the presence of calcium
phosphate crystals, and calcium phosphate precipitates readily in the presence
of uric acid crystals.

Question: 59
The most common cause of Humoral hypercalcemia of malignancy
(HHM) is :

A PTH

B Ectopic production of
Vitamin D

C PTHrP

D PGE2

33.78% People got this right

Explanation:

Correct Answer (C) Humoral hypercalcemia of malignancy (HHM) occurs in up to


20% of patients with cancer. HHM is most common in cancers of the lung, head
and neck, skin, esophagus, breast, and genitourinary tract and in multiple
myeloma and lymphomas. There are several distinct humoral causes of HHM,
but it is caused most commonly by overproduction of PTHrP.

Question: 60
Among the following which is not an absolute indication for performing
dialysis in acute renal failure?

A Refractory Fluid overload

https://emedicoz.com/testresult/683510s14692528 47/160
6/9/24, 12:52 PM Neet PG Preparation, Neet PG Coaching, FMGE, USMLE

B Refractory Hyperkalemia

C Pericardial rub

D Serum creatinine > 6 mg/dl

21.79% People got this right

Explanation:

Correct Answer (D) Serum Creatinine > 6mg/dl Absolute indications for dialysis
are: Uremic pericarditis/pleuritis/encephalopathy/coagulopathy, Refractory Fluid
overload, Refractory Metabolic Acidosis, Severe Azotemia > 100.

Question: 61
Which one of the following investigations is not required for evaluation
of recurrent pregnancy losses?

A TORCH titres

B Karyotype of an abortus

C aPTT

D A and B

20.23% People got this right

Explanation:

Correct Answer (D)

A and B

https://emedicoz.com/testresult/683510s14692528 48/160
6/9/24, 12:52 PM Neet PG Preparation, Neet PG Coaching, FMGE, USMLE

Infections can cause solo abortion but these cannot cause recurrent pregnancy
losses. Karyotype of couple is necessary to rule out genetic cause of RPL e.g.
translocations. Fetal karyotype is not needed since it does not provide any
important information. aPTT is done to evaluate APLA syndrome.

Question: 62
Which one of the following is not a clinical criterion for the diagnosis of
APLA syndrome?

A 3 or more abortions of
more than 10 weeks

B Arterial thrombosis

C Venous thrombosis

D Early onset pre-eclampsia


leading to preterm delivery of less
than 34 weeks

20.48% People got this right

Explanation:

Correct Answer (A)

3 or more abortions of more than 10 weeks

https://emedicoz.com/testresult/683510s14692528 49/160
6/9/24, 12:52 PM Neet PG Preparation, Neet PG Coaching, FMGE, USMLE

Question: 63
Which one of the following cannot cross the placental barrier?

A Insulin

B Heparin

C IgM

D All of the above

50.25% People got this right

Explanation:

Correct Answer (D)

All of the above

All are large molecules and hence these molecules cannot cross the placenta.

Question: 64
A 25 years old multiparous patient has Rh incompatibility. Which one of
the following is least possible in this case?
https://emedicoz.com/testresult/683510s14692528 50/160
6/9/24, 12:52 PM Neet PG Preparation, Neet PG Coaching, FMGE, USMLE

A Neonatal jaundice

B Oligohydramnios

C Fetal anemia

D Placentomegaly

41.32% People got this right

Explanation:

Correct Answer (B) Oligohydramnios Due to antigen- antibody reaction, there is


increased fluid formation in the amniotic fluid by the placenta which is enlarged
due to neovascularization. This leads to polyhydramnios and not
oligohydramnios.

Question: 65
A 22 years old Primigravida has 36 weeks of gestation with twin
pregnancy. Which one of the following is true about this pregnancy?

A History of twinning is
common on the paternal side

B Most common type of


twins will be DCDA

C Post term pregnancy is a


known complication

D Cesarean section is done if


the presentation is cephalo-
transverse

https://emedicoz.com/testresult/683510s14692528 51/160
6/9/24, 12:52 PM Neet PG Preparation, Neet PG Coaching, FMGE, USMLE

48.12% People got this right

Explanation:

Correct Answer (B) Most common type of twins will be DCDA 70-80% of the
twins are dizygotics. All the dizygotics are dichorionics diamniotics. Twinning
history is on the maternal side. Vaginal delivery is done if the first baby is in
cephalic presentation. Preterm delivery is common and post term is not seen.

Question: 66
Identify the following instrument?

A Allis tissue holding forceps

B Babcock’s tissue holding


forceps

C Kocher’s clamp

D Maingot’s clamp

58.19% People got this right

Explanation:

https://emedicoz.com/testresult/683510s14692528 52/160
6/9/24, 12:52 PM Neet PG Preparation, Neet PG Coaching, FMGE, USMLE

Correct Answer (B) Babcock’s tissue holding forceps These are used to hold
tubular structures like fallopian tube, round ligament etc. These are atraumatic
forceps hence these are used to hold the bladder and intestines.

Question: 67
Which one of the following is an incorrect pair of cause and effect?

A Ebstein’s anomaly –
polyhydramnios

B Potter’s syndrome –
oligohydramnios

C Potter’s sequence – key


hole sign

D Indomethacin – Patent
ductus arteriosus

24.88% People got this right

Explanation:

Correct Answer (D)

Indomethacin – Patent ductus arteriosus Indomethacin is NSAID.

It reduces prostaglandins. It may cause premature closure of ductus arteriosus in


fetus. It does not cause PDA.

Question: 68
Which one of the following is used to calculate the amount of injectable
iron dose?

https://emedicoz.com/testresult/683510s14692528 53/160
6/9/24, 12:52 PM Neet PG Preparation, Neet PG Coaching, FMGE, USMLE

A (2.4 x Wt in pounds x Hb
deficit) +500mg

B (2.4 x Wt in pounds x Hb of
the patient) +500mg

C (2.4 x Wt in Kg x Hb deficit)
+500mg

D (2.4 x Wt in kg x Hb of the
patient) +500mg

56.8% People got this right

Explanation:

Correct Answer (C) (2.4 x Wt in Kg x Hb deficit) +500mg The total iron deficit is
calculated using the Ganzoni formula :- Total Iron Deficit = Weight × (Target Hb
in g/dL - Actual Hb in g/dL) × 2.4 + Iron Stores Note: 500 if W > 35kg 15 mg/kg
if W < 35kg

Question: 69
During the treatment of diabetes in pregnancy, what should be the target
blood sugar levels to be achieved?

A F =<92, 1hr =<180, 2hr =


<153mg%

B F =<95, 1hr =<140, 2hr =


<120mg%

C F =<92, 1hr =<140, 2hr =


<153mg%

https://emedicoz.com/testresult/683510s14692528 54/160
6/9/24, 12:52 PM Neet PG Preparation, Neet PG Coaching, FMGE, USMLE

D F =<95, 1hr =<180, 2hr =


<140mg%

27.42% People got this right

Explanation:

Correct Answer (B) F =<95, 1hr =<140, 2hr =<120mg% This is as per ADA.

Question: 70
The ideal time to switch over from warfarin to heparin in a pregnant
patient with history of valve replacement surgery in past is?

A 32 weeks

B 36 weeks

C During labour

D After delivery

48.42% People got this right

Explanation:

Correct Answer (B) 36 weeks Warfarin is a long acting anti-coagulant. It requires


1 week to get out of the body after stopping the pills. Warfarin causes PPH if
patient goes in labour. This is the reason why it should be stopped at 36 weeks.
Heparin is started immediately.

Question: 71
Which one of the following is not the effect of magnesium sulphate
toxicity?

https://emedicoz.com/testresult/683510s14692528 55/160
6/9/24, 12:52 PM Neet PG Preparation, Neet PG Coaching, FMGE, USMLE

A DTR 1+

B Urine output <100 cc in 4


hours

C Respiratory depression

D Pontine paralysis

13.6% People got this right

Explanation:

Correct Answer (B) Brine output <100 cc in 4 hours MgSO4 is a CNS depressant.
The earliest sign of toxicity is blunting of DTR i.e. +1. It does not cause reduction
of urine output but pre-eclampsia and eclampsia can reduce urine output which
will actually cause retention of MgSO4 in the body.

Question: 72
Which one of the following is not done in the treatment of variable
deceleration in labour?

A Propped up position

B PV examination

C Cysto-distension

D Amnio-infusion

13.93% People got this right

Explanation:

https://emedicoz.com/testresult/683510s14692528 56/160
6/9/24, 12:52 PM Neet PG Preparation, Neet PG Coaching, FMGE, USMLE

Correct Answer (A) Propped up position Variable decelerations are due to


umbilical cord compression. This may be due to cord prolapse as well. It requires
head-low position in order to avoid cord compression by fetal head. PV
examination is done to rule out cord prolapse. If there is cord- prolapse then
manual disengagement or bladder distension is done. If there is no cord-
prolapse then amnioinfusion is done.

Question: 73
If vaginal cytology is done in a post-menopausal patient, it will show
predominance of which cells?

A Superficial

B Intermediate

C Navicular

D Basal

45.93% People got this right

Explanation:

Correct Answer (D) Basal Superficial cells are eosinophilic cells which are
predominantly seen if estrogen is high. Intermediate cells a.k.a. navicular cells
are predominant if progesterone is high. Basal and parabasal cells are
predominant if patient is post-menopausal due to lack of estrogen and
progesterone.

Question: 74
Which one of the following conditions will have presence of Mullerian as
well as Wolffian ducts?

https://emedicoz.com/testresult/683510s14692528 57/160
6/9/24, 12:52 PM Neet PG Preparation, Neet PG Coaching, FMGE, USMLE

A AIS

B SWYER syndrome

C 46XY with AMH receptor


deficiency

D Savage syndrome

34.43% People got this right

Explanation:

Correct Answer (C) 46XY with AMH receptor deficiency AIS has no MULLERIAN
due to Sertoli cells and no WOLFFIAN due to lack of androgen receptors. SWYER
has gonadal dysgenesis so there are no Sertoli cells so MULLERIAN ducts are
present. Savage syndrome is a female with resistant FSH receptors. MULLERIAN
ducts are present. Male with AMH receptor deficiency will have MULLERIAN
ducts along with WOLFFIAN ducts.

Question: 75
Which of the following is not a correct pair of organism and antibiotic
used?

A Trichomonas vaginalis –
metronidazole

B Candidiasis- fluconazole

C Herpes genitalis – imiferon

D Neisseria gonorrhoea -
ceftriaxone

https://emedicoz.com/testresult/683510s14692528 58/160
6/9/24, 12:52 PM Neet PG Preparation, Neet PG Coaching, FMGE, USMLE

59.48% People got this right

Explanation:

Correct Answer (C) Herpes genitalis – imiferon Protozoa and anaerobes are
treated by metronidazole. Fungal infection is treated by fluconazole. Herpes is
treated by acyclovir. Neisseria is treated by 3rd gen cephalosporin.

Question: 76
Which symptom is not seen in a case of fibroid uterus?

A Spasmodic dysmenorrhea

B Dyspareunia

C Lump in abdomen

D Loin pain due to


hydronephrosis

38.94% People got this right

Explanation:

Correct Answer (B) Dyspareunia Fibroids cause spasmodic as well as congestive


dysmenorrhea. If fibroid uterus is more than 12 weeks sized then there is a
palpable lump in abdomen. Large fibroids can compress ureters and hence
hydronephrosis is possible. Dyspareunia is not a feature of fibroid.

Question: 77
A 72 years old post-menopausal patient has bleeding per vaginum since
last 3 months. On speculum examination, there was 1 cm sized growth on
the posterior lip of cervix. Which one of the following is the next best line
of management?

https://emedicoz.com/testresult/683510s14692528 59/160
6/9/24, 12:52 PM Neet PG Preparation, Neet PG Coaching, FMGE, USMLE

A PAP smear

B Colposcopy guided biopsy

C 4 quadrant biopsies

D Direct punch biopsy

34.87% People got this right

Explanation:

Correct Answer (D) Direct punch biopsy Visible cervical lesion requires direst
punch biopsy of the lesion. It would be a confirmatory test.

Question: 78
Cervical cancer with parametrial involvement along with hydronephrosis
due to ureteric obstruction and bullous edema of bladder is considered
as?

A Stage 2B

B Stage 3B

C Stage 4A

D Stage 4B

33.64% People got this right

Explanation:

Correct Answer (B)

Stage 3B
https://emedicoz.com/testresult/683510s14692528 60/160
6/9/24, 12:52 PM Neet PG Preparation, Neet PG Coaching, FMGE, USMLE

Question: 79
The earliest change that occurs in puberty is?

A Thelarche

B Pubarche

C Peak growth velocity

D Menarche

41.97% People got this right

Explanation:

Correct Answer (A) Thelarche Out of these 4 options, thelarche is the earliest
change which occurs on an average at 9-10 years of age. If there would have
been growth spurt in the options then that would have been the correct answer.
Peak growth velocity is not the same. It occurs at the time of adrenarche.

Question: 80
Which one of the following is not an indication of starting Hormonal
replacement therapy?

https://emedicoz.com/testresult/683510s14692528 61/160
6/9/24, 12:52 PM Neet PG Preparation, Neet PG Coaching, FMGE, USMLE

A After gonadectomy in AIS

B Premature menopause

C Prevention of
cardiovascular disease

D Hot flushes

55.65% People got this right

Explanation:

Correct Answer (C) Prevention of cardiovascular disease CVS diseases may


increase due to thromboembolic effect of estrogen. Estrogen increases HDL
cholesterol but CVS diseases are not prevented.

Question: 81
Which one of the following is a risk factor in the development of cervical
cancer?

A Nulliparous patient

B COC pills

C HPV 6 infection

D None of the above

16.14% People got this right

Explanation:

Correct Answer (B) COC pills COC pills taken for more than 5 years may slightly
increase the risk of adenocarcinoma of cervix. HPV 6 is a benign virus and it
https://emedicoz.com/testresult/683510s14692528 62/160
6/9/24, 12:52 PM Neet PG Preparation, Neet PG Coaching, FMGE, USMLE

causes genital warts. It is not the risk factor.

Question: 82
Which one of the following is true regarding barrier contraceptives?

A Best method of
contraceptive in a lactating
mother

B Prevention of bacterial
vaginosis

C Highly effective
contraceptive method

D None of the above

23.02% People got this right

Explanation:

Correct Answer (B) Prevention of bacterial vaginosis Bacterial vaginosis is more


common in patients with STI. Barrier contraceptives prevent STI and hence these
prevent bacterial vaginosis.

Question: 83
MIRENA is a hormonal IUD. It contains which generation progesterone?

A 1st

B 2nd

https://emedicoz.com/testresult/683510s14692528 63/160
6/9/24, 12:52 PM Neet PG Preparation, Neet PG Coaching, FMGE, USMLE

C 3rd

D 4th

31.21% People got this right

Explanation:

Correct Answer (B) 2nd MIRENA contains levonorgestrel - 52 mg. LNG is a


second-generation progesterone.

Question: 84
Identify the correct match:

A VVF – Martius flap surgery

B Uretero vaginal fistula –


Boari flap surgery

C SUI – Burch
colposuspension

D All of the above

40.96% People got this right

Explanation:

Correct Answer (D) All of the above

Question: 85
In a young patient of 25 years of age with uterine prolapse, SHIRODKAR’s
surgery is done as the treatment. In that surgery, mersilene tape is used

https://emedicoz.com/testresult/683510s14692528 64/160
6/9/24, 12:52 PM Neet PG Preparation, Neet PG Coaching, FMGE, USMLE

to recreate the function of which ligament?

A Mackenrodt’s ligament

B Pubocervical ligament

C Uterosacral ligament

D Sacrospinous ligament

34.22% People got this right

Explanation:

Correct Answer (C) Uterosacral ligament In Shirodkar’s sling surgery, mersilene


tape is attached from isthmus of uterus to sacrum. It works like uterosacral
ligament and lifts the prolapsed uterus.

Question: 86
Ochronosis occurs due to accumulation of

A Phenylpyruvate

B Xanthurenate

C Glyoxylate

D Homogentisic acid

44.13% People got this right

Explanation:

https://emedicoz.com/testresult/683510s14692528 65/160
6/9/24, 12:52 PM Neet PG Preparation, Neet PG Coaching, FMGE, USMLE

Correct Answer (D) Homogentisic acid

Question: 87
Not true regarding Fish oil

A Rich in omega 3 fatty acids

B Rich in Eicosapentenoic
acid

C Used in management of
type II hyperlipidemia

D Used in management of
hypertriglyceridemia

28.13% People got this right

Explanation:

Correct Answer (C) Used in management of type II hyperlipidemia

Question: 88
Activity of pyruvate carboxylase depend on which of the following
positive allosteric effector

A Acetyl CoA

B AMP

C Isocitrate

https://emedicoz.com/testresult/683510s14692528 66/160
6/9/24, 12:52 PM Neet PG Preparation, Neet PG Coaching, FMGE, USMLE

D Succinate

29.6% People got this right

Explanation:

Correct Answer (A) Acetyl CoA

Question: 89
In an experimental setup to demonstrate oxygen uptake , pyruvate and
malate was added to mitochondria,which resulted in increased oxygen
uptake. On addition of a unknown compound the ATP production
declined drastically with increased oxygen demand and heat production ?
Which of the following is the unknown substance?

A Rotenone

B 2,4, Dinitrophenol

C Oligomycin

D Antimycin

29.98% People got this right

Explanation:

Correct Answer (B) 2,4, Dinitrophenol

Question: 90
Which of the following is an example of post translational modification

https://emedicoz.com/testresult/683510s14692528 67/160
6/9/24, 12:52 PM Neet PG Preparation, Neet PG Coaching, FMGE, USMLE

A N-Glycosylation of proteins

B 7-methyl guanosine
capping

C Poly A tail addition

D γ-carboxylation of
glutamate residues

18.79% People got this right

Explanation:

Correct Answer (D) γ-carboxylation of glutamate residues

Question: 91
Triple phase CT liver includes all except

A Portal venous phase

B Arterial phase

C Capillary phase

D Parenchymal phase

36.81% People got this right

Explanation:

Correct Answer (C) Capillary phase

https://emedicoz.com/testresult/683510s14692528 68/160
6/9/24, 12:52 PM Neet PG Preparation, Neet PG Coaching, FMGE, USMLE

Question: 92
Patient presents to emergency with RTA and suspected blunt abdominal
trauma. FAST was performed and image is shown below. His vitals 110/70,
pulse 70/min. What's next best step?

A Diagnostic peritoneal
lavage

B CECT

C Exploratory laparotomy

D Nothing to be done

39.4% People got this right

Explanation:

Correct Answer (B) CECT

Question: 93
What's the most likely diagnosis?

https://emedicoz.com/testresult/683510s14692528 69/160
6/9/24, 12:52 PM Neet PG Preparation, Neet PG Coaching, FMGE, USMLE

A Child Abuse

B Rickets

C Scurvy

D Ochronosis

42.49% People got this right

Explanation:

Correct Answer (B)

Rickets

Imaging findings

o Cupping and fraying of metaphysis

o Poorly mineralized epiphyseal centers with delayed appearance

o Irregular widened epiphyseal plates (increased osteoid)

o Increase in distance between end of shaft and epiphyseal center

o Cortical spurs projecting at right angles to metaphysis

o Coarse trabeculation (not the ground-glass pattern found in scurvy)

o Periosteal reaction may be present

https://emedicoz.com/testresult/683510s14692528 70/160
6/9/24, 12:52 PM Neet PG Preparation, Neet PG Coaching, FMGE, USMLE

o Deformities common

Bowing of long bones


Molding of epiphysis
Fractures
Frontal bossing

Question: 94
Non-contrast CT Brain: Sudden onset severe headache. What does the
image show?

A Subarachnoid
haemorrhage

B Intraventricular
haemorrhage

C Extradural haematoma

D Intracerebral haemorrhage

52.65% People got this right

Explanation:

Correct Answer (A) SAH

https://emedicoz.com/testresult/683510s14692528 71/160
6/9/24, 12:52 PM Neet PG Preparation, Neet PG Coaching, FMGE, USMLE

Question: 95
Personal dose monitoring is done by?

A TLD badge

B Linear accelerator

C Gamma Camera

D GM counters

50.27% People got this right

Explanation:

Correct Answer (A) TLD badge Thermoluminescent dosimeter (TLD) is a passive


radiation detection device that is used for personal dose monitoring or to
measure patient dose.

Question: 96
Same saturation in all cardiac chambers is seen in which disorder

A TGA

B TAPVC

C Tricuspid atresia

D TOF

42.87% People got this right

Explanation:
https://emedicoz.com/testresult/683510s14692528 72/160
6/9/24, 12:52 PM Neet PG Preparation, Neet PG Coaching, FMGE, USMLE

Correct Answer (B)

TAPVC

Question: 97
Most common urinary tract obstruction in males

A Posterior urethral valves

B Anterior urethral valves

C Ureterocele

D VUR

54.78% People got this right

Explanation:

Correct Answer (A)

Posterior urethral valves

https://emedicoz.com/testresult/683510s14692528 73/160
6/9/24, 12:52 PM Neet PG Preparation, Neet PG Coaching, FMGE, USMLE

Question: 98
A 14 days old neonates developed conjunctivitis followed by pneumonitis
showing bilateral infiltrates. Peripheral blood smear shows lymphocytic
predominance. Diagnosis:

A Pseudomonas

B Chlamydia

C Mycoplasma

D Streptococcus

41.23% People got this right

Explanation:

Correct Answer (B)

Chlamydia

https://emedicoz.com/testresult/683510s14692528 74/160
6/9/24, 12:52 PM Neet PG Preparation, Neet PG Coaching, FMGE, USMLE

Question: 99
A 11 months old boy presents with recurrent bloody stools, atopic
dermatitis, petechiae and platelet count of 20,000/mm3. What is the next
step?

A Screen for WASP gene

B Bone marrow biopsy

C Check for antibodies


against platelets

D LFT & serum creatinine

45.77% People got this right

Explanation:

Correct Answer (A)

WASP gene

https://emedicoz.com/testresult/683510s14692528 75/160
6/9/24, 12:52 PM Neet PG Preparation, Neet PG Coaching, FMGE, USMLE

Question: 100
ACTH is the drug of choice for

A Tuberous sclerosis

B West syndrome

C Rolandic epilepsy

D JME

42.41% People got this right

Explanation:

Correct Answer (B) West syndrome or Infantile spasms- the drug of choice is
ACTH

Question: 101
A child reports to you at 1 year of age. He has received DPT at 6 weeks.
What should be the next step?

https://emedicoz.com/testresult/683510s14692528 76/160
6/9/24, 12:52 PM Neet PG Preparation, Neet PG Coaching, FMGE, USMLE

A DPT full course repeat

B Only DT

C Next dose DPT 2nd dose

D No need of DPT now

44.51% People got this right

Explanation:

Correct Answer (C)

Next dose DPT

Question: 102
Tongue fasciculations are seen in

A Myasthenia gravis

B Spinal Muscular Atrophy

https://emedicoz.com/testresult/683510s14692528 77/160
6/9/24, 12:52 PM Neet PG Preparation, Neet PG Coaching, FMGE, USMLE

C Duchenne muscular
dystrophy

D GBS

40.91% People got this right

Explanation:

Correct Answer (B) SMA is a disease of spinal motor neuron, and has all features
of lower motor neuron disease including fasciculations.

Question: 103
A 10 years old boy presented with fatigue and joint pains. He has a rash as
shown in the picture. What is your diagnosis?

A Juvenile RA

B Juvenile Dermatomyositis

C Osteoarthritis

D Sclerodactyly

56.39% People got this right

https://emedicoz.com/testresult/683510s14692528 78/160
6/9/24, 12:52 PM Neet PG Preparation, Neet PG Coaching, FMGE, USMLE

Explanation:

Correct Answer (B)

Juvenile dermatomyositis. The child has typical gottron papules.

Diagnostic criteria of Juvenile dermatomyositis are shown in this figure

Question: 104
A 4 years old boy of 15kg, 100cms tall has blood urea 100mg/dl and
serum creatinine 1mg/dl Calculate eGFR

A 80 ml/min/1.73m2

B 40 ml/min/1.73m2

C 33 ml/min/1.73m2

D 55 ml/min/1.73m2

11.11% People got this right

Explanation:

https://emedicoz.com/testresult/683510s14692528 79/160
6/9/24, 12:52 PM Neet PG Preparation, Neet PG Coaching, FMGE, USMLE

Correct Answer (B) 40 Schwartz formula for eGFR- eGFR= k x height in cms ------
--------------------- serum creatinine (mg/dl) Remember= 0.413 Enzymatic
method

Question: 105
A child has myoclonus jerk and decreased school performance. He was
well till now. 12 years ago, he had fever & rash (at age of 1 years). What is
most likely diagnosis?

A CSF IgG Measles

B MRI for Mesial temporal


sclerosis

C AQ4 antibodies

D CECT brain

35.69% People got this right

Explanation:

Correct Answer (A)

CSF IgG measles

SSPE

• Early onset of Measles: 50%<2 yrs age, 75%<4 yrs age

• Lag 7-13 years

• Stage 1 → Stage 2 → Stage 3 → Stage 4

• School performance → Myoclonus → Choreoathetosis → Death

https://emedicoz.com/testresult/683510s14692528 80/160
6/9/24, 12:52 PM Neet PG Preparation, Neet PG Coaching, FMGE, USMLE

Question: 106
Find out the true and false statements A. Drug Transport across the cell
membrane is mainly by active transport- B. Alcohol is eliminated from
body by 1st order kinetic- C. In competitive inhibition potency decreases
but efficacy remain same D. therapeutic index indicates safety of the drug
E. Continuous exposure of a receptor to antagonist results in down
regulation of receptor

A A-F, B-F, C-T, D-T, E-F

B A-T, B-T, C-T, D-T, E-F

C A-F, B-T, C-F, D-F, E-T

D A-T, B-F, C-T, D-T, E-T

26% People got this right

Explanation:

Correct Answer (A) Drug Transport across the cell membrane is mainly by passive
diffusion Alcohol is eliminated from body by zero order kinetic- In competitive
inhibition potency decreases but efficacy remain same therapeutic index
indicates safety of the drug Continuous exposure of a receptor to antagonist
results in up regulation of receptors

Question: 107
Which anti thyroid drug causes the problem shown in the images

https://emedicoz.com/testresult/683510s14692528 81/160
6/9/24, 12:52 PM Neet PG Preparation, Neet PG Coaching, FMGE, USMLE

A Propylthiouracil

B Methimazole

C Lugol's iodine

D Radioactive iodine I131

44.51% People got this right

Explanation:

Correct Answer (B) Methimazole causes skin rashes and agranulocytosis and
causes teratogenicity in the form of aplastic cutis and choanal atresia
Propylthiouracil causes severe hepatotoxicity and it is relatively safe in
pregnancy

Question: 108
Find out the true statement regarding action of drug mentioned at the site
of arrow mark mentioned in the urinary bladder

https://emedicoz.com/testresult/683510s14692528 82/160
6/9/24, 12:52 PM Neet PG Preparation, Neet PG Coaching, FMGE, USMLE

A Mirabegran acting as a
beta 3 stimulant useful in
managing post operative urinary
retention

B Oxybutynin acting M3
stimulant useful in overactive
bladder

C Bethanechol acting as M3
stimulant useful in managing
urinary retention

D Tamsulosin acting as a
alpha 1 antagonist, useful in BPH

27.33% People got this right

Explanation:

Correct Answer (C) Mirabegran acting as a beta 3 stimulant in the bladder,


causes detrusor relaxation thereby useful in managing overactive bladder
Oxybutynin acting M3 antagonist useful in overactive bladder Bethanechol
acting as M3 stimulant useful in managing urinary retention Tamsulosin acting as
a alpha 1 antagonist, useful in BPH. Tamsulosin acting on internal urethral
sphincter not in the bladder

https://emedicoz.com/testresult/683510s14692528 83/160
6/9/24, 12:52 PM Neet PG Preparation, Neet PG Coaching, FMGE, USMLE

Question: 109
Regarding drug useful in clotting disorder, find out the drugs which are
safe in renal failure? 1. Heparin 2. Enoxaparin 3. Bivalirudin 4. Dabigatran
5. Abciximab

A 1&2

B 2&3

C 3&4

D 1&5

14.17% People got this right

Explanation:

Correct Answer (D)

Question: 110
Which of the following thiazide is effective in patient who has
hypertension with eGFR<30?

A Dhlorthalidone

https://emedicoz.com/testresult/683510s14692528 84/160
6/9/24, 12:52 PM Neet PG Preparation, Neet PG Coaching, FMGE, USMLE

B Metalazone

C Hydrochlorthiazide

D Chlorthiazide

24.8% People got this right

Explanation:

Correct Answer (B) In patients with chronic renal failure, thiazide diuretics are
ineffective. But metolazone, despite being a thiazide diuretic, is effective as
antihypertensives in patients with chronic renal failure.Metolazone is useful even
in severe renal failure (GFR of ~ 15 mL/min). Thiazides are the diuretics of choice
for uncomplicated hypertension. Mechanisms of antihypertensive action:
Decreases plasma and ECF volume by diuresis thereby decreasing cardiac output
• Even when the compensatory mechanisms restore the Na+ levels and plasma
volume, a low BP is maintained by a reduction in total peripheral resistance. •
This reduction in total peripheral resistance is due to the persistence of a small
sodium and volume deficit even after compensation. The decreased intracellular
sodium is responsible for increased compliance of the vessel wall.

Question: 111
All of the opioid analgesic has additionally 5HT reuptake inhibitory action
except

A Pethidine

B Buprenorphine

C Tramadol

D Tapentadol

16.38% People got this right


https://emedicoz.com/testresult/683510s14692528 85/160
6/9/24, 12:52 PM Neet PG Preparation, Neet PG Coaching, FMGE, USMLE

Explanation:

Correct Answer (B) Meperidine to block neuronal reuptake of 5HT, resulting in


serotonergic overactivity Tapentadol is structurally and mechanistically similar to
tramadol. It is a weak inhibitor of monoamine reuptake SERT inhibition by
tramadol, tapentadol, methadone, dextromethorphan and pethidine may
contribute to the serotonin syndrome

Question: 112
Which one of the following drugs useful in gastric ulcer will cause adverse
effects of megaloblastic anemia?

A Cimetidine

B Colloidal bismuth sub


citrate

C Omeprazole

D Sucralfate

26.95% People got this right

Explanation:

Correct Answer (C) PPIs- The most common side effects are nausea, abdominal
pain, constipation, flatulence, and diarrhea. Subacute myopathy, arthralgias,
headaches, interstitial nephritis, and skin rashes also have been reported.
Chronic treatment with omeprazole decreases the absorption of vitamin B12, but
the clinical relevance of this effect is not clear. Chronic use of PPIs has been
reported to be associated with an increased risk of bone fracture and with
increased susceptibility to certain infections (e.g., hospital-acquired pneumonia,
community-acquired Clostridium difficile, spontaneous bacterial peritonitis in
patients with ascites). Hypergastrinemia is more frequent and more severe with
PPIs than with H2 receptor antagonists and associated with this is ECL
hyperplasia, fundic gland polyposis, and atrophic gastritis. Recently, there have
been associations made between long-term PPI use and increased risk of
chronic kidney disease and dementia. H2 antagonist- Side effects are minor and

https://emedicoz.com/testresult/683510s14692528 86/160
6/9/24, 12:52 PM Neet PG Preparation, Neet PG Coaching, FMGE, USMLE

include diarrhea, headache, drowsiness, fatigue, muscular pain, and constipation.


Less-common side effects include those affecting the CNS (confusion, delirium,
hallucinations, slurred speech, and headaches), which occur primarily with
intravenous administration of the drugs or in elderly subjects. Several reports
have associated H2 receptor antagonists with various blood disorders, including
thrombocytopenia. H2 receptor antagonists cross the placenta and are excreted
in breast milk. Although no major teratogenic risk has been associated with
these agents, caution is warranted when they are used in pregnancy The most
common side effect of sucralfate is constipation, The “sticky” nature of the
viscous gel produced by sucralfate in the stomach also may be responsible for
the development of bezoars in some patients

Question: 113
Which one of the following drugs has been recently approved for the
treatment of HIV infection?

A Letermovir

B Fostemsavir

C Tecovirimat

D Sofosbuvir

21.79% People got this right

Explanation:

Correct Answer (B) Fostemsavir has been recently approved (2020) by FDA for
the treatment of HIV-1 (human immunodeficiency virus-1) infection. Fostemsavir
is an HIV-1 gp120-directed attachment inhibitor. It is available as 60Omg
extended- release tablets. The recommended dosage is one tablet taken twice
daily with or without food. Letermovir is an antiviral drug for the treatment
of cytomegalovirus (CMV) infections. It has been tested in CMV infected patients
with allogeneic stem cell transplants and may also be useful for other patients
with a compromised immune system such as those with organ transplants
or HIV infections Tecovirimat, is an antiviral medication with activity against
orthopoxviruses such as smallpox and monkeypox. It is the first antipoxviral
https://emedicoz.com/testresult/683510s14692528 87/160
6/9/24, 12:52 PM Neet PG Preparation, Neet PG Coaching, FMGE, USMLE

drug approved in the United States. It is an inhibitor of the orthopoxvirus VP37


envelope wrapping protein. Sofosbuvir is an antiviral medicine that is used to
treat chronic hepatitis C in adults and children who are at least 3 years old.

Question: 114
Which of the following anti TB drugs acting by inhibiting the site mentioned
in the arrow mark?

A Pyrazinamide

B Ethambutol

C Delamanid

D Bedaquiline

20.51% People got this right

Explanation:

Correct Answer (B) iNH, pyrazinamide and delamanid acts by inhibiting mycolic
acid synthesis ethambutol acts by inhibiting arabinogalacton synthesis
Bedaquiline blocks the proton pump for ATP synthase of mycobacteria

https://emedicoz.com/testresult/683510s14692528 88/160
6/9/24, 12:52 PM Neet PG Preparation, Neet PG Coaching, FMGE, USMLE

Question: 115
Match the following, regarding anticancer drugs and their toxicity and
toxicity amelioration.

1. P-D- V

2. Q- A- V

3. R- B- Z

4. S- C- X

5. S- B- Y

6. Q- D-V

A 1, 2, 3 are correct match

B 2, 3, 4 are correct match

C 3 & 4 are correct match

D 5 & 6 are correct match

24.66% People got this right

Explanation:

Correct Answer (D)

https://emedicoz.com/testresult/683510s14692528 89/160
6/9/24, 12:52 PM Neet PG Preparation, Neet PG Coaching, FMGE, USMLE

Question: 116
A 48-year-old woman notices a firm, fixed mass in her right breast but
chooses not to seek medical care. The mass is then detected during her
annual physical examination 8 months later. By that point, it has
increased greatly in size, causing her discomfort. When asked why she did
not report the mass when she first noticed it, she says, "I was waiting for
my children to graduate from high school. I didn't want to have a bunch
of doctors' appointments when I should be helping them with their
homework." This response is an example of which of the following
defense mechanisms?

A Denial

B Reaction formation

C Rationalization

D Displacement

43.23% People got this right

Explanation:

Correct Answer (C)

Rationalization

https://emedicoz.com/testresult/683510s14692528 90/160
6/9/24, 12:52 PM Neet PG Preparation, Neet PG Coaching, FMGE, USMLE

Rationalization is an immature defense mechanism characterized by excusing an


unacceptable behavior (or emotion) in a false but logically rational way to avoid
the true reasons for the behavior. Immature defense mechanisms are typically
maladaptive and prevent patients from coping with difficult issues in a
productive manner. This patient was frightened about potential pain from cancer
treatment and so used her children's school work as a justification for avoiding
the proper evaluations. This resulted in a delay in care and likely a more difficult
treatment course.

(Choice A) Denial, an immature defense mechanism, is the failure to accept a


disturbing aspect of external reality. An example would be if this patient refused
to acknowledge that a potentially malignant mass is present despite obvious
appearances or other evidence of cancer.

Question: 117
A 60-year-old physician is admitted to the hospital for jaundice and
weight loss. He has no other symptoms, including abdominal pain and
changes in bowel movements. His past medical history is non-
contributory. The patient has no history of significant alcohol intake but
does have a 30-pack-year smoking history. After a detailed workup,
advanced pancreatic carcinoma is diagnosed. Shortly after discharge, he
obtains his medical records and spends several hours a day analysing the
details of his diagnostic tests and researching the chemical composition
of proposed chemotherapy regimens. But his family members report he
seems otherwise oddly unconcerned and detached about his poor
prognosis. Which of the following psychological defense mechanisms is
this patient most likely employing?

https://emedicoz.com/testresult/683510s14692528 91/160
6/9/24, 12:52 PM Neet PG Preparation, Neet PG Coaching, FMGE, USMLE

A Denial

B Intellectualization

C Isolation of affect

D Rationalization

25.89% People got this right

Explanation:

Correct Answer (B) Intellectualization Psychological defense mechanisms are


unconscious means of responding and adapting to emotional stressors while
preserving one's self-image. They serve to decrease anxiety, minimize feelings of
shame and vulnerability, provide a sense of security or control, and insulate a
person from disappointments. They are classified as immature (including
"primitive" and "neurotic" defense mechanisms) or mature. Immature defense
mechanisms are more commonly used by children or adolescents and prevent
the individual from coping with reality. Certain immature defense mechanisms
are fairly common in adults and provide short-term relief, but they often result
in long-term complications. Mature defense mechanisms are most often used by
well-adjusted adults and allow for "healthy" and constructive adaptations to
reality. This patient spends hours reviewing his medical records and dwelling on
the small details of the workup and treatment. He is employing
intellectualization, a defense mechanism that allows him to avoid anxiety by
focusing on the technical aspects of his cancer diagnosis while ignoring the
personal ramifications of it. Intellectualization is generally considered an
immature or neurotic defense mechanism involving repression of the emotional
component of a stressor. But in some cases it can lead to more adaptive coping
strategies as patients channel their intellectual efforts in productive ways.
(Choice A) Denial is the failure to accept a disturbing aspect of external reality.
An example in this patient would be refusal to acknowledge his diagnosis and
insistence that nothing is wrong in the face of all evidence to the contrary.
(Choice C) Isolation of affect involves separating a thought from its emotional
components, thereby making it tolerable. An example would be this patient
receiving news of his diagnosis and speaking without emotion to his distraught
family about his poor prognosis and likely death. (Choice D) Rationalization
involves attempting to justify or excuse a behavior or decision rather than
acknowledge the significance or consequences of it. An example would be this

https://emedicoz.com/testresult/683510s14692528 92/160
6/9/24, 12:52 PM Neet PG Preparation, Neet PG Coaching, FMGE, USMLE

patient justifying not seeking treatment by saying that it would make no


difference in his prognosis. This explanation would allow him to exert more
control over his situation and not admit that anxiety and fear likely prevented
him from seeking help earlier.

Question: 118
A smoker Is worried about the side effects of smoking. But he does not
stop smoking thinking that he smokes less as compared to other and
takes a good diet . This thinking is callsed as

A Self exemption

B Cognitive behavior

C Self-protection

D Distortion

52.43% People got this right

Explanation:

Correct Answer (A) Self exemption is seen in pre contemplation stage, where the
person exempts himself of side effects of the respective substance. (eg : an
alcoholic saying that, he cant have so many adverse effects, bcoz he has strong
LIVER)

Question: 119
Which of the following is a sleep disturbance characteristic of mania?

A Reduced early morning


sleep

https://emedicoz.com/testresult/683510s14692528 93/160
6/9/24, 12:52 PM Neet PG Preparation, Neet PG Coaching, FMGE, USMLE

B Reduced initial sleep

C Reduced need for sleep

D Reduced latency of sleep

49.84% People got this right

Explanation:

Correct Answer (C) Both mania and depression are characterized by various
neurovegetative signs – disturbances in sleep, appetite, weight, energy levels,
and circadian functions. In depression early morning awakening is characteristic.
To qualify as early morning awakening, a patient must wake up at least 2 hours
prior to his usual waking time. In mania, there is increased energy associated
with reduced need for sleep. Patients can go on for many days with barely any
sleep. Paradoxically, sleep deprivation itself can induce a state similar to
hypomania in some susceptible individuals.

Question: 120
Carphologia is seen in:

A Delirium

B Dementia

C Tourette syndrome

D Ganser's syndrome

25.31% People got this right

Explanation:

Correct Answer (A) Delirium (Ref : Niraj Ahuja) The motor symptoms in delirium
can include: 1. Asterixis (flapping tremor), 2. Multifocal myoclonus, 3.
https://emedicoz.com/testresult/683510s14692528 94/160
6/9/24, 12:52 PM Neet PG Preparation, Neet PG Coaching, FMGE, USMLE

Carphologia or floccillation (picking movements at cover-sheets and clothes), 4.


Occupational delirium (elaborate pantomimes as if continuing their usual
occupation in the hospital bed), and 5. Tone and reflex abnormalities.

Question: 121
A female patient presents with diffuse alopecia to you. She had suffered
from typhoid fever 3 months back. Most probable diagnosis is:

A Androgenetic alopecia

B Telogen effluvium

C Anagen effluvium

D Alopecia areata

55.3% People got this right

Explanation:

Correct Answer (B)

Question: 122
A farmer having trauma in lower limb following which a cauliflower like
growth developed on that site. On HPE Copper penny bodies are seen.
What is the diagnosis?

https://emedicoz.com/testresult/683510s14692528 95/160
6/9/24, 12:52 PM Neet PG Preparation, Neet PG Coaching, FMGE, USMLE

A Chromoblastomycosis

B Mycetoma

C Sporotrichosis

D Pheohyphomycosis

59.09% People got this right

Explanation:

Correct Answer (A)

Question: 123
Identify the condition shown in the image ?

https://emedicoz.com/testresult/683510s14692528 96/160
6/9/24, 12:52 PM Neet PG Preparation, Neet PG Coaching, FMGE, USMLE

A Condyloma acuminata

B Bowen disease

C Condyloma lata

D Hemorrhoids

51.8% People got this right

Explanation:

Correct Answer (A)

Question: 124
A patient presented with following nail changes and HPE shows collection
of neutrophils in epidermis. Diagnosis?

A Psoriasis

B Lichen planus

C Alopecia areata

https://emedicoz.com/testresult/683510s14692528 97/160
6/9/24, 12:52 PM Neet PG Preparation, Neet PG Coaching, FMGE, USMLE

D Atopic dermatits

50.87% People got this right

Explanation:

Correct Answer (A)

Question: 125
A 27 year old patient was diagnosed to have borderline leprosy and
started on multibacillary MDT. Six weeks later he developed pain in the
nerves and redness and swelling of the skin lesins. The management of
his illness should include all of the following except?

A Stop antileprosy drugs

B Systemic corticosteroids

C Rest to limb affected

D Analgesic

49.89% People got this right

Explanation:

Correct Answer (A)

Question: 126
Which of the following hormones is secreted by the anterior pituitary?

https://emedicoz.com/testresult/683510s14692528 98/160
6/9/24, 12:52 PM Neet PG Preparation, Neet PG Coaching, FMGE, USMLE

A Growth hormone releasing


hormone

B Dopamine

C Gonadotropin releasing
hormone

D Thyroid stimulating
hormone

49.62% People got this right

Explanation:

Correct Answer (D) Thyroid stimulating hormone TSH is secreted by the


thyrotropes in the anterior pituitary. GHRH, Dopamine or PIF and GnRH are all
secreted by the hypothalamus.

Question: 127
CO2 generated in the tissues is carried in the venous blood primarily as
which of the following?

A CO2 in the plasma

B H2CO3 in the plasma

C HCO3- in the plasma

D CO2 in the RBCs

37.96% People got this right

Explanation:
https://emedicoz.com/testresult/683510s14692528 99/160
6/9/24, 12:52 PM Neet PG Preparation, Neet PG Coaching, FMGE, USMLE

Correct Answer (C) HCO3- in the plasma CO2 generated in the tissues enters the
venous blood, and in the RBCs, combines with H2O in the presence of carbonic
anhydrase to form H2CO3. H2CO3 dissociates into H+ and HCO3-. The H+
remains in the RBCs to be buffered by the deoxyhemoglobin, and the
bicarbonate moves into the plasma in exchange for Cl-. Thus, CO2 is carried in
the venous blood to the lungs as bicarbonates. In the lungs, the reactions occur
in reverse: CO2 is generated and expired.

Question: 128
Which of the following would cause an increase in GFR?

A Constriction of the afferent


arteriole

B Constriction of the efferent


arteriole

C Constriction of the ureter

D Increased plasma protein


concentration

53.5% People got this right

Explanation:

Correct Answer (B) Constriction of the efferent arteriole Constriction of the


efferent arteriole creates a back pressure, increasing the hydrostatic pressure in
the glomerular capillaries (PGC) and therefore, the GFR. Constriction of the
afferent arteriole decreases glomerular capillary flow decreasing the PGC and
therefore, decreasing the GFR. Constriction of the ureter creates a backpressure
increasing the PBC. PBC opposes GFR. Increase in PBC therefore, decreases GFR.
Increasing the plasma protein concentration increases the plasma colloid
osmotic pressure. Plasma colloid osmotic pressure opposes GFR. Increase in the
plasma colloid osmotic pressure decreases GFR.

https://emedicoz.com/testresult/683510s14692528 100/160
6/9/24, 12:52 PM Neet PG Preparation, Neet PG Coaching, FMGE, USMLE

Question: 129
A 20-year-old man is brought to the Emergency after being injured in an
automobile accident and sustaining significant blood loss. He is given a
transfusion of 3 units to stabilize his blood pressure. Before the
transfusion, which of the following is true about his condition?

A His total peripheral


resistance was decreased

B His heart rate was


decreased

C Firing rate of his carotid


sinus nerves was increased

D Sympathetic outflow to his


heart and blood vessels was
increased

37.66% People got this right

Explanation:

Correct Answer (D) Sympathetic outflow to his heart and blood vessels was
increased Blood loss results in decreased firing rate of the baroreceptors. This in
turn results in an increase in the sympathetic outflow to the heart and blood
vessels. This in turn increases the heart rate, total peripheral resistance, and
blood pressure.

Question: 130
Which of the following gastrointestinal secretions is hypotonic, has a high
HCO3- content, and has its production inhibited by vagotomy?

A Saliva

https://emedicoz.com/testresult/683510s14692528 101/160
6/9/24, 12:52 PM Neet PG Preparation, Neet PG Coaching, FMGE, USMLE

B Gastric secretion

C Pancreatic secretion

D Bile

13.24% People got this right

Explanation:

Correct Answer (A) Saliva Saliva is hypotonic and has a high HCO3- content.
Vagal stimulation increases saliva production. Vagotomy (or atropine) will
decrease the secretion of saliva producing a dry mouth.

Question: 131
A 35-year-old homeless man comes to the emergency department due to
urinary urgency, burning sensation on urination, and urethral discharge. The
patient had similar symptoms 3 months ago. and urethral swab microscopy
revealed numerous neutrophils and gram-negative diplococci as shown in
the image. Choose the correct selective medium for this pathogen.

A PNF medium

B Cetrimide agar

https://emedicoz.com/testresult/683510s14692528 102/160
6/9/24, 12:52 PM Neet PG Preparation, Neet PG Coaching, FMGE, USMLE

C Ashdown medium

D New York city Medium

26.87% People got this right

Explanation:

Correct Answer (D) It’s a case of gonococcal urethritis. Gram smear shows Gram
negative, intracellular, kidney shaped diplococci. Selective medium is New York
city medium, Mod. Thayer Martin medium and GC Lect medium. Stuart medium
is a transport medium for gonococci.

Question: 132
A 48-year-old man comes to the physician due to 3 days of fever. shortness
of breath, pleuritic chest pain, and cough productive of green sputum. He
has smoked a pack of cigarettes daily for 20 years. On examination. there
are crackles at the base of the left lung. Chest x-ray reveals a left lower-lobe
consolidation. Sputum gram smear shown in the image. Which of the
following characteristics are these bacteria likely to demonstrate?

A Bacitracin sensitivity

B Bile solubility

https://emedicoz.com/testresult/683510s14692528 103/160
6/9/24, 12:52 PM Neet PG Preparation, Neet PG Coaching, FMGE, USMLE

C Growth in hypertonic saline

D Optochin resistance

21.68% People got this right

Explanation:

Correct Answer (B) The presence of lanceolate-shaped, Gram-positive diplococci


in the sputum sample of a patient with fever and cough is suggestive of
infection with Streptococcus pneumoniae. All streptococci are catalase-negative.
The degree of hemolysis on blood agar distinguishes hemolytic (partial or
complete) from nonhemolytic streptococci. The bile solubility and optochin tests
are used to differentiate between partial {green} hemolytic species alpha -
hemolysis namely S pneumoniae and viridans streptococci group. Autolysin
produced by S pneumoniae is activated by bile, resulting in cell lysis. Therefore.
addition of bile salts into a tube with S pneumoniae growth (turbid) leads to cell
lysis (clearing of turbidity), this is known as bile solubility. S pneumoniae is bile-
soluble and optochin—sensitive; viridans group streptococci are bile-insoluble
and optochin resistant

Question: 133
A 34-year-old man comes to the office due to a painless penile ulcer.
which he first noticed 3 clays ago. He had unprotected sexual intercourse
with a new partner a few weeks ago. The patient has no significant
medical history and takes no medications. Examination reveals a 2-cm
nontender ulcer close to the glans penis with a raised. indurated margin
and a clean base. There is no surrounding lesions or vesicles. There are
several bilateral enlarged inguinal lymph nodes. which are firm,
nontender, and rubbery. Physical examination is otherwise unremarkable.
Rapid plasma regain and HIV testing are negative. Infection with which of
the following is the most likely cause of this patient's symptoms?

A Haemophilus ducreyi

B Herpes simplex virus

https://emedicoz.com/testresult/683510s14692528 104/160
6/9/24, 12:52 PM Neet PG Preparation, Neet PG Coaching, FMGE, USMLE

C Neisseria gonorrhoeae

D Treponema pallidum

44.4% People got this right

Explanation:

Correct Answer (D) This patient developed a genital ulcer a few weeks after
unprotected intercourse. raising Suspicion for a sexually transmitted infection. A
wide range of pathogens can cause genital ulcers. but a solitary. painless ulcer
with heaped-up borders and a clean base usually indicates primary syphilis
(syphilitic chancre}. Syphilis is caused by Treponema pallidum. a gram-negative
spirochete that cannot be cultured and cannot be detected on Gram stain due to
its small size. The diagnosis of syphilis relies primarily on nontreponemal (eg.
rapid plasma regain. VDRL) and treponemal (eg. fluorescent treponemal
antibody absorption) serologic testing. However. because humoral antibody
response often takes 4 weeks to develop. false-negative serologic testing is
common early in the course of disease Nontreponemal tests are particularly
susceptible to initial false-negative results (as in this patient).

Question: 134
A 23-year-old woman participates in a research study evaluating the
effect of monoclonal antibodies on viral infection. The patient has no past
medical history and takes no medications. She has not been ill recently.
During the study. a peripheral blood specimen is obtained. CD19 positive
cells are isolated. purified. and exposed to monoclonal antibodies against
cell surface complement receptor CD21. The cells are subsequently
incubated along with several viruses being studied. Initial exposure to
monoclonal antibodies against CD21 is most likely to prevent cell
infection with which of the following viruses?

A Adenovirus

B Cytomegalovirus

https://emedicoz.com/testresult/683510s14692528 105/160
6/9/24, 12:52 PM Neet PG Preparation, Neet PG Coaching, FMGE, USMLE

C Epstein-Barr virus

D Parvovirus B19

34.76% People got this right

Explanation:

Correct Answer (C) The Initial attachment of the virion envelope or capsid
surface proteins to the complementary host cell surface receptors is essential to
viral tropism for specific tissues and invasion of cells. Many viruses bind to
normal host cell plasma membrane receptors to enter host cells. Epstein-Barr
virus (EBV) is a herpesvirus responsible for acute infectious mononucleosis.
nasopharyngeal carcinoma. and certain lymphomas (eg Burkitt lymphoma), more
than 90% of the normal adult population is seropositive for EBV, which is
primarily transmitted through contact with oropharyngeal secretions. The EBV
envelope glycoprotein gp350 binds to CD21 (also known as CR2) the cellular
receptor for the C3d complement component. CD21 is normally present on the
surface of B cells (CD19-positive cells) and nasopharyngeal epithelial cells.
Therefore, exposure to a monoclonal anti-CD21 antibody could interfere with
EBV attachment to B cells.

Question: 135
A 28-year-old G1P0 woman at 16 weeks estimated gestational age
presents for prenatal care. Routine prenatal screening tests are performed
and reveal a positive HIV antibody test. The patient is extremely
concerned about the possible transmission of HIV to her baby and wants
to know if this occurred as soon as possible after delivery. Which of the
following would be the most appropriate diagnostic test to address this
patient‘s concern?

A Antigen assay for P24

B Viral culture

C CD4+ T cell count

https://emedicoz.com/testresult/683510s14692528 106/160
6/9/24, 12:52 PM Neet PG Preparation, Neet PG Coaching, FMGE, USMLE

D HIV DNA PCR

43.8% People got this right

Explanation:

Correct Answer (D) HIV DNA PCR is the diagnostic test of choice to detect HIV
infection in neonates / infants born to HIV positive mothers. It is a rapid test and
needs only a small dried spot of blood initially for screening. If positive. the test
is repeated with a whole blood sample to confirm the diagnosis. If the initial PCR
is negative, a repeat test may be needed at 6 months if the baby is breastfed as
the potential for transmission persists.

Question: 136
A 25-year-old man with multiple injuries sustained in a motorcycle
accident develops osteomyelitis while in the hospital The organism is
identified as methicillin-sensitive Staphylococcus aureus: and antibiotics
are started. Which of the following is the most important measure to
reduce the risk of transmission to other patients?

A Contact precautions

B Hand hygiene

C Isolation precautions

D Masks

41.15% People got this right

Explanation:

Correct Answer (B) Hand hygiene by health care workers is the single most
important measure to reduce the risk of transmission of microorganisms
between patients. It includes handwashing with soap and water or using alcohol-
based hand sanitizers that do not require water. Proper handwashing technique
involves washing all surfaces of the hands and fingers with soap and water for at
https://emedicoz.com/testresult/683510s14692528 107/160
6/9/24, 12:52 PM Neet PG Preparation, Neet PG Coaching, FMGE, USMLE

least 15 seconds, drying with a disposable towel. and using the towel to turn off
the faucet. Alcohol-based hand sanitizer should cover all surfaces of the hands
and fingers and be allowed to dry. The World Health Organization recommends
handwashing before and after touching a patient, before aseptic procedures,
and after contact with body fluids or patient surroundings

Question: 137
A 38-year-old man is undergoing treatment for acute myelogenous
leukemia. initially. he complained of fevers and being lethargic and fatigued
all the time. However. of late. he has noted increasingly severe right-sided
headaches. He describes the pain to be throbbing in nature and located
behind his right eye, which is also associated with nasal stuffiness. The
patient denies any previous history of allergies. Physical examination reveals
right sided proptosis and periorbital tenderness. Biopsy of the right
maxillary sinus mucosa shown in the image.

A Malassezia furfur

B Microsporum canis

C Rhizopus species

D Aspergillus fumigatus

35.55% People got this right

https://emedicoz.com/testresult/683510s14692528 108/160
6/9/24, 12:52 PM Neet PG Preparation, Neet PG Coaching, FMGE, USMLE

Explanation:

Correct Answer (C) Mucor. Rhizopus. and Absidia species are saprophytic fungi
present in the environment. They are transmitted by spore inhalation and cause
Mucormycosis. Mucormycosis is very strongly associated with diabetic
ketoacidosis, Patients with underlying immunosuppression due to solid organ
transplantation, hematologic malignancies. or glucocorticoid therapy are also at
high risk. Mucormycosis tends to affect the paranasal sinuses. Patients complain
of facial and periorbital pain, headache, and purulent nasal discharge. The fungi
proliferate in the walls of blood vessels and cause necrosis of the corresponding
tissue. Black eschar (necrotic tissue) may be seen on the palate or nasal
turbinates. Mucormycosis is diagnosed by light microscopy of a tissue specimen.
Broad aseptate hyphae with vertical branching.

Question: 138
Quality control Agent for cold sterilization is

A Brevundimonas dimunita

B Bacillus pumilus

C Geobacillus
stearothermophilus

D Bacillus atropheus

22.28% People got this right

Explanation:

Correct Answer (B) Cold sterilization is gamma radiation – Ionizing radiation.


QCA – Bacillus pumilus.

Question: 139

https://emedicoz.com/testresult/683510s14692528 109/160
6/9/24, 12:52 PM Neet PG Preparation, Neet PG Coaching, FMGE, USMLE

A renal transplant was rejected by the host within few minutes after the
transplantation. It belongs to which type of hypersensitivity

A Type I HSR

B Type II HSR

C Type III HSR

D Type IV HSR

37.68% People got this right

Explanation:

Correct Answer (B) Hyperacute rejection – with in few minutes Due to pre
existing antibodies in the host. Belongs to type II HSR. Complement mediated
Lysis.

Question: 140
All the following helminths are inhabiting in the bile duct of humans
except

A Fasciola hepatica

B Clonorchis sinensis

C Opisthorchis viverrine

D Fasciolapsis buski

24.3% People got this right

https://emedicoz.com/testresult/683510s14692528 110/160
6/9/24, 12:52 PM Neet PG Preparation, Neet PG Coaching, FMGE, USMLE

Explanation:

Correct Answer (D) F. buski is intestinal fluke. All the other options are liver
flukes and residing in human bile duct. C. sinensis causes Cholangio adeno
carcinoma.

Question: 141
What is not true for the area marked in the given pic is

A Its area of increased


fluoroscence

B Abnormal leaky vessels


lead to this

C Seen in early stages of


Diabetic retinopathy

D Regression with treatment


is a good prognostic sign

40.8% People got this right

Explanation:

Correct Answer (C) Marked area is neovascularization elsewhere in left retina.


They are abnormal vessels and leaks dye. It is seen in proliferative stage of

https://emedicoz.com/testresult/683510s14692528 111/160
6/9/24, 12:52 PM Neet PG Preparation, Neet PG Coaching, FMGE, USMLE

diabetic retinopathy and regresses on treatment with panretinal


photocoagulation or anti vegf therapy (good prognosis)

Question: 142
An ophthalmologist wants to check fundus of a myopic patient
(-4Dioptres). He prefers increased magnification over field of view Which
condensing lens should he use?

A 30 D

B 28 D

C 20 D

D 14 D

13.82% People got this right

Explanation:

Correct Answer (D) Magnification with Fundoscopy = Power of eye /power of


condensing lens Less the power of condensing lens, more the magnification and
lesser the field of view

Question: 143
A 4-year-old girl presents with acute, unilateral, left-sided periocular pain
and proptosis, Which condition would not be included in the differential
diagnosis?

A Optic glioma

B Ruptured dermoid

https://emedicoz.com/testresult/683510s14692528 112/160
6/9/24, 12:52 PM Neet PG Preparation, Neet PG Coaching, FMGE, USMLE

C Retinoblastoma

D Chocolate cyst

6.72% People got this right

Explanation:

Correct Answer (A) Ruptured dermoid and retinoblastoma can both present with
rapidly developing proptosis. Orbital lymphangioma may produce acute
proptosis when there is a hemorrhage(chocolate cyst)

Question: 144
MRI head and orbit revealed an infarct in the pons. Which of the following
muscles will be affected due to it?

A A

B B

C C

D D

41.45% People got this right


https://emedicoz.com/testresult/683510s14692528 113/160
6/9/24, 12:52 PM Neet PG Preparation, Neet PG Coaching, FMGE, USMLE

Explanation:

Correct Answer (A) A.Lateral rectus B.Superior oblique C.Inferior oblique


D.Medial rectus Lateral rectus is supplied by 6th nerve whose nucleus is in pons

Question: 145
Which of the following is false for the visual field defect shown here?

A The visual field shown here


is of right eye

B Arcuate fibres of retina are


affected

C Seen in optic neuropathies

D Central field is affected

26.13% People got this right

Explanation:

Correct Answer (B) Right visual field showing centro-ceacal scotoma


Papillomacular bundle fibres are affected

https://emedicoz.com/testresult/683510s14692528 114/160
6/9/24, 12:52 PM Neet PG Preparation, Neet PG Coaching, FMGE, USMLE

Question: 146
Which of the following is spherical equivalent for the prescription +2DS
-2DC ×90 degrees?

A +2DS

B +1DC

C +1DS

D 0 DS

19.52% People got this right

Explanation:

Correct Answer (C) Spherical power + (cylinder power)/2

Question: 147
A 33 year old male came with pain and watering in the right eye for 36
hours. On examination a 3x2 cm corneal ulcer is seen with elevated
margins , feathery hyphae, finger like projections and minimal hypopyon
in cornea. What is not true among the following

A Corneal scraping should be


done for detecting organism

B Intraocular pressure with


Goldmann applanation should be
checked

C Topical antibiotics is given


on an hourly basis initially

https://emedicoz.com/testresult/683510s14692528 115/160
6/9/24, 12:52 PM Neet PG Preparation, Neet PG Coaching, FMGE, USMLE

D Non responding ulcer will


need therapeutic keratoplasty

30.2% People got this right

Explanation:

Correct Answer (B) Fungal corneal ulcer Symptoms. Gradual onset of pain,
grittiness, photophobia, blurred vision and watery or mucopurulent discharge.
Grey or yellow–white stromal infiltrate with indistinct fluffy margins. Progressive
infiltration, often with satellite lesions. Feathery branch-like extensions or a ring-
shaped infiltrate may develop. Rapid progression with necrosis and thinning can
occur. Goldmann applanation is a contact procedure to cornea, which should
be avoided in a patient with active corneal ulcer

Question: 148
In a patient of Neurosyphillis, what is seen

A Pupil reacts to light and


near object

B Pupil reacts to light but not


to near object

C Pupil does not react to


light and near object

D Pupil does not react to


light but reacts to near object

37.98% People got this right

Explanation:

https://emedicoz.com/testresult/683510s14692528 116/160
6/9/24, 12:52 PM Neet PG Preparation, Neet PG Coaching, FMGE, USMLE

Correct Answer (D) Neurosyphillis affects pretectal nucleus and cause Argyll
Roberston Pupil, where light near dissociation is seen. It affects the pupillary
light reflex pathway but spares the more ventral pupillary near reflex pathway.

Question: 149
A patient has u/l ptosis and difficulty in eye movement. A drug was given
and ptosis recovered within 5 mins. What is the condition.

A Myasthenia gravis

B Myotonic dystrophy

C 3rd nerve palsy

D Aponeurotic ptosis

54.53% People got this right

Explanation:

Correct Answer (A) Tensilon Test for myasthenia gravis Edrophonium


hydrochloride: Anti-acetylcholinesterase (e.g. increases acetylcholine effects at
neuromuscular junction) and improves ptosisIce pack test also improves ptosis

Question: 150
The glasses in the pic can be given in which type of squint

https://emedicoz.com/testresult/683510s14692528 117/160
6/9/24, 12:52 PM Neet PG Preparation, Neet PG Coaching, FMGE, USMLE

A Refractive accommodative
esotropia

B Non-refractive
accommodative esotropia

C Infantile esotropia

D Sensory esotropia

13% People got this right

Explanation:

Correct Answer (B) Accommodative esotropia This esotropia is associated with


an onset after 1 year of age. They often have a gradual onset and may evolve to
becoming constant after a period of intermittency. Refractive esotropia is
associated with high hyperopia of +3.00 D and corrected by hypermetropia
glasses. Non refractive/ convergence excess type esotropia is associated with a
deviation greater at near than distance. The deviation at near is reduced when
accommodation is relaxed with +3.00D lenses. The mechanism is an abnormal
ratio between accommodative convergence and accommodation (high
accommodative convergence to accommodation ratio); the effort to
accommodate elicits an abnormally high accommodative convergence response.
Mixed accomodative esotropia is a residual esotropia that is partially corrected,
yet still persists in spite of full correction of hyperopic refractive error.

https://emedicoz.com/testresult/683510s14692528 118/160
6/9/24, 12:52 PM Neet PG Preparation, Neet PG Coaching, FMGE, USMLE

Question: 151
Which of the following set is correctly matched for purpose of these
indicators:

A A-III B-II C-I

B A-II B-III C-I

C A-I B-II C-III

D A-III B-I C-II

55.41% People got this right

Explanation:

Correct Answer (D) Case fatality rate :Severity of disease Proportional mortality
rate : Burden of disease 5 year survival rate : Prognosis

Question: 152
A new chemoprophylactic regimen has been introduced. Because of this
new prophylactic intervention :

A Only incidence will change

https://emedicoz.com/testresult/683510s14692528 119/160
6/9/24, 12:52 PM Neet PG Preparation, Neet PG Coaching, FMGE, USMLE

B Both incidence and


prevalence will change

C Neither incidence nor


prevalence will change

D Only prevalence will


change

27.23% People got this right

Explanation:

Correct Answer (B) Relationship between Prevalence, Incidence and Duration: P


= I x D Larger the duration, greater the prevalence, shorter the duration (due to
recovery or death), lower the prevalence, compared to incidence rate. •
Improvement in treatment – may ↓ duration & hence ↓ prevalence • If Rx
prevents death but doesn’t produce recovery, prevalence will increase. • If
duration is ↓ed sufficiently, prevalence could take place despite ↑ in incidence. •
For chronic diseases – Increase in incidence  Increase in prevalence • For easily
curable or fatal diseases – Increases in incidence  Prevalence may remain same
or even decreases as duration is decreased.

Question: 153
Transition from increased prevalence of infectious pandemic diseases to
manmade disease is known as

A Paradoxical transition

B Reversal of transition

C Epidemiological transition

D Demographic transition

https://emedicoz.com/testresult/683510s14692528 120/160
6/9/24, 12:52 PM Neet PG Preparation, Neet PG Coaching, FMGE, USMLE

30.61% People got this right

Explanation:

Correct Answer (C) Epidemiological transition : • Transition from increased


prevalence of infectious pandemic diseases to manmade disease . • Referred as
secular trends • Long term change over decades

Question: 154
Organism multiplying and developing in the hosts is called as :

A Cyclopropagative

B Cyclodevelopmental

C Developmental

D Propogative

33.37% People got this right

Explanation:

Correct Answer (A) Biological transmission: • Propagative: Mutiplication of agent


Ex: plague bacilli in rat fleas,Yellow fever in aedes • Cyclo-propagative:
Mutiplication and devt of agent Ex: Malarial parasite in anopheles • Cyclo-
developmental: Devt of agent Ex: filarial parasite in culex, guineaworm in
Cyclops.

Question: 155
SPIKES protocol is related to :

A Ethics in trials

https://emedicoz.com/testresult/683510s14692528 121/160
6/9/24, 12:52 PM Neet PG Preparation, Neet PG Coaching, FMGE, USMLE

B To disclose bad news

C Hospital welfare

D Public private partnership

39.92% People got this right

Explanation:

Correct Answer (B) SPIKES is an acronym for presenting distressing information


in an organized manner to patients and families. The SPIKES protocol provides a
step-wise framework for difficult discussions such as when cancer recurs or when
palliative or hospice care is indicated. Each letter represents a phase in the six-
step sequence. S stands for setting, P for perception, I for invitation or
information, K for knowledge, E for empathy, and S for summarize or strategize.

Question: 156
When a drug is evaluated for its usefulness in real life community
settings:

A Effectiveness

B Efficacy

C Usefulness

D Yield

30.23% People got this right

Explanation:

Correct Answer (A) Efficacy pertains to the desired effect created by a procedure,
or service. It is based on the extent to which that intervention results in the

https://emedicoz.com/testresult/683510s14692528 122/160
6/9/24, 12:52 PM Neet PG Preparation, Neet PG Coaching, FMGE, USMLE

effect researchers want to see, assuming the ideal conditions (like a controlled
enviroment in a lab). Effectiveness is about real-world scenarios. It focuses on
uncontrolled circumstances, such as those that occur outside the laboratory. It is
based on the extent to which goals are achieved as a result of the intervention,
whatever that might be

Question: 157
President of NIDM is :

A Health minister

B Home Minister

C Prime minister

D President of India

18.68% People got this right

Explanation:

Correct Answer (B) • Disaster Management : Under Ministry of Home affairs •


National institute of disaster management (NIDM) : New Delhi • National
institute of disaster management (NIDM) : President – Home minister • National
disaster management authority : Chairman – Prime minister • First response
during disaster : from district level

Question: 158
Polio virus recently eradicated :

https://emedicoz.com/testresult/683510s14692528 123/160
6/9/24, 12:52 PM Neet PG Preparation, Neet PG Coaching, FMGE, USMLE

A WPV-1

B WPV-2

C WPV-3

D All of these

11.71% People got this right

Explanation:

Correct Answer (C) Polio virus recently eradicated is : WPV-3 WPV 2 and WPV 3
has been eradicated

Question: 159
Identify the image shown below :

A Forest plot

B ROC curve

C Kaplan meir curve

https://emedicoz.com/testresult/683510s14692528 124/160
6/9/24, 12:52 PM Neet PG Preparation, Neet PG Coaching, FMGE, USMLE

D None of these

44.59% People got this right

Explanation:

Correct Answer (B) ROC curve : used to assess performance of a diagnostic test

Question: 160
Neonatal screening for hypothyroidism is a type of :

A Prospective screening

B Prescriptive screening

C Rehabilitation

D Diagnostic method

31.81% People got this right

Explanation:

Correct Answer (B)

https://emedicoz.com/testresult/683510s14692528 125/160
6/9/24, 12:52 PM Neet PG Preparation, Neet PG Coaching, FMGE, USMLE

Question: 161
Global hunger index considers all of the following except :

A Proportion of
undernourished population

B Proportion of underweight
children ( less than 5yrs )

C Child mortality

D Maternal mortality

53.39% People got this right

Explanation:

Correct Answer (D)

Global hunder index :

Question: 162
Vector shown below can cause all except :

https://emedicoz.com/testresult/683510s14692528 126/160
6/9/24, 12:52 PM Neet PG Preparation, Neet PG Coaching, FMGE, USMLE

A Kala azar

B Sand fly fever

C Oriental sore

D Plantar ulcer

42.74% People got this right

Explanation:

Correct Answer (D)

Plantar ulcer is by sand flea

https://emedicoz.com/testresult/683510s14692528 127/160
6/9/24, 12:52 PM Neet PG Preparation, Neet PG Coaching, FMGE, USMLE

Question: 163
A systematic observation and recording of activities of one or more
individuals at random interval is done in

A Systems analysis

B Network analysis.

C Work sampling

D Input-output analysis

29.22% People got this right

Explanation:

Correct Answer (C)

Question: 164
GATHER approach is used for :

A Counselling a client

https://emedicoz.com/testresult/683510s14692528 128/160
6/9/24, 12:52 PM Neet PG Preparation, Neet PG Coaching, FMGE, USMLE

B Treating NCDs

C Awareness among students

D Covid prevention

31.02% People got this right

Explanation:

Correct Answer (A) GATHER approach : Ex : To counsel regarding family planning.


G – Greet the client A – Ask/ascertain – needs/problems T – Telling different
methods/options to solve problem H – Help to make voluntary decision E –
Explain fully the chosen decision/action R – Return for follow up visit

Question: 165
Which of the following is an ultimate desired state :

A Goal

B Objective

C Target

D Indicator

46.72% People got this right

Explanation:

Correct Answer (A)

https://emedicoz.com/testresult/683510s14692528 129/160
6/9/24, 12:52 PM Neet PG Preparation, Neet PG Coaching, FMGE, USMLE

Question: 166
Following an emergency caesarean section under general anaesthesia, the
patient presents with respiratory distress and tachycardia in recovery.
Auscultation reveals coarse crepitations in right lower lobe. Her condition
is most likely due to:

A Endotracheal intubation.

B Positive pressure
ventilation

C Mendelson syndrome

D Extubation of the patient in


lateral recumbent position with
lowered head.

35.8% People got this right

Explanation:

Correct Answer (C) • Aspiration is the most common cause of maternal


anaesthesia-related mortality. • The risk of aspiration pneumonitis can be
decreased by reducing the volume and acidity of the gastric contents. • The
gastric emptying time is prolonged in labour. • Extubation should be performed
with the patient fully awake. • Patients who aspirate may develop clinical signs
and symptoms several hours after the incident. • Aspiration is potentially a risk in

https://emedicoz.com/testresult/683510s14692528 130/160
6/9/24, 12:52 PM Neet PG Preparation, Neet PG Coaching, FMGE, USMLE

all patients with reduced consciousness. • The treatment is mainly supportive. • It


includes oxygen therapy and bronchodilators. • It may often be necessary to
remove large particulate matter using bronchoscopy. • Use of prophylactic
antibiotics and steroids is not advised • CPAP or intermittent positive pressure
ventilation with PEEP may be necessary in severe cases.

Question: 167
A 25-year-old primipara has an accidental Dural tap (ADT) with a 16-G
Tuohy needle while the anaesthetist was attempting labour epidural
analgesia. A day after the ADT, the patient is complaining of severe
positional frontal and occipital headache. Which of the following is not
appropriate as first line of management?

A Prophylactic bed rest.

B Simple analgesics.

C Caffeine

D Epidural blood patch.

30.17% People got this right

Explanation:

Correct Answer (D) There is no evidence for prophylactic bed rest in post-dural
puncture headache. All the other mentioned treatments have some benefit, with
the blood patch having the best results. Management of PDPH Conservative
management approaches include: • bed rest • encouraging intake of oral fl uids
and/or intravenous hydration • reassurance. Pharmacological approaches
include: • caffeine — either intravenous (e.g. 500 mg caff eine in 1 L saline) or
orally • regular analgesia: paracetamol, diclofenac etc. • 5HT agonists
(e.g.sumatriptan) Interventional approaches include: • epidural blood patch,
which involves injecting approximately 20 mL of the patient’s own fresh blood
(taken in a strict sterile fashion) into the epidural space near the site of the
suspected puncture. • It is successful in most cases, and the onset of relief from
headache may be immediate (but occasionally takes up to 24 h). • In patients in

https://emedicoz.com/testresult/683510s14692528 131/160
6/9/24, 12:52 PM Neet PG Preparation, Neet PG Coaching, FMGE, USMLE

whom a blood patch is not successful, or where relief is temporary, it may be


repeated, although the likelihood of improvement is reduced.

Question: 168
After finding an unresponsive child while walking through the park, and
confirming the child isn't breathing what would be your next course of
action?

A Leave the child and search


for an AED

B Deliver rescue breaths as


most cardiac arrest occur due to
breathing problems

C Begin back blows and


chest thrusts

D Deliver 30 chest
compressions followed by 2
breaths for 2 mins and then leave
the child to call for help.

37.08% People got this right

Explanation:

Correct Answer (D)

https://emedicoz.com/testresult/683510s14692528 132/160
6/9/24, 12:52 PM Neet PG Preparation, Neet PG Coaching, FMGE, USMLE

Question: 169
Which of the following flowmeter arrangements would be the most likely
to lead to a hypoxic gas mixture?

A N2O, Air, O2

B Air, N2O, O2

C N2O, O2, Air

D O2, Air, N2O

25.42% People got this right

Explanation:

Correct Answer (D) • The arrangement of flowmeters must be such that a


hypoxic mixture does not result in the event of a leak. • Hypoxic mixtures are
most likely to occur when nitrous is the most downstream flowmeter and least
likely to occur when the oxygen flowmeter is located downstream from both the
air and nitrous oxide flowmeters. • A hypoxic mixture is less likely when oxygen
is furthest downstream because nitrous would also advance the oxygen toward
the common gas outlet.

https://emedicoz.com/testresult/683510s14692528 133/160
6/9/24, 12:52 PM Neet PG Preparation, Neet PG Coaching, FMGE, USMLE

Question: 170
Which of the following would cause an abrupt decrease in the end-tidal
CO2 (ETCO2) during general anesthesia?

A Hyperthermia

B Sepsis

C Shivering

D Pulmonary embolism

51.2% People got this right

Explanation:

Correct Answer (D)

• An abrupt decrease in the ETCO2 is often linked with a life-threatening


predicament and is almost always associated with either a ventilation or
perfusion issue.
• Examples include sudden severe hypotension, endotracheal tube migration,
massive pulmonary embolism, and cardiac arrest.
• When an abrupt decrease in ETCO2 is noted, it should be quickly verified and
addressed.
• During cardiopulmonary resuscitation, adequacy of circulation can be
monitored via return of the ETCO2.

https://emedicoz.com/testresult/683510s14692528 134/160
6/9/24, 12:52 PM Neet PG Preparation, Neet PG Coaching, FMGE, USMLE

Question: 171
In a rape trial, where it is already accepted that sexual intercourse
happened between a male and a female, the proof of consent of sexual
intercourse is upon?

A Male

B Female

C Judge

D Witness

28.15% People got this right

Explanation:

Correct Answer (A) Sec 114 A IEA (Indian Evidence Act)- once the sexual
intercourse between a male and A female is accepted in the court and the query
is whether it happened with the Consent of the said female or not and the
female in her statement says that she Did not consent. The court has to believe
her statement and the male has to prove That he had sexual intercourse with her
with her consent.

https://emedicoz.com/testresult/683510s14692528 135/160
6/9/24, 12:52 PM Neet PG Preparation, Neet PG Coaching, FMGE, USMLE

Question: 172
Chromolacryorrhea is seen in?

A Corrosive poisoning

B Organo-phosphorus
poisoning

C Heavy metal poisoning

D Ophitoxemia

21.6% People got this right

Explanation:

Correct Answer (B)

Question: 173
Dr Shivam is a prosecution witness. He tells the court that the cause of
death in a case is hemorrhagic shock due to cutting of abdominal aorta
by a knife. He is acting in the court as?

A Hostile witness

B Simple witness

C Expert witness

D Amicus curie

62.32% People got this right

https://emedicoz.com/testresult/683510s14692528 136/160
6/9/24, 12:52 PM Neet PG Preparation, Neet PG Coaching, FMGE, USMLE

Explanation:

Correct Answer (C) Expert witness- one who is expert in his field of work
(autopsy surgeon). Can give Opinion based upon his observation in his felid of
work

Question: 174
Judicial hanging is an example of which type of hanging?

A Partial + typical

B Complete + atypical

C Partial + atypical

D Complete + typical

31.84% People got this right

Explanation:

Correct Answer (B) Complete hanging- the body is completely suspending in the
air. Complete weight is Upon the rope. Partial hanging- some part of body is
touching the ground, so only partial weight is upon Rope. Typical hanging- the
knot of the ligature mark is at the back of the neck. Atypical hanging- the knot
of the ligature mark is not at the back of the neck.

Question: 175
The photograph shows the appearance of skull after opening Cranial vault.
The hemorrhage seen is?

https://emedicoz.com/testresult/683510s14692528 137/160
6/9/24, 12:52 PM Neet PG Preparation, Neet PG Coaching, FMGE, USMLE

A EDH

B SDH

C SAH

D INTRACEREBRAL BLEED

53.44% People got this right

Explanation:

Correct Answer (A) After opening cranial vault, the dura matter is visible. The
hemorrhage is present on Top of it thus EDH. The frequency of hemorrhages
found at autopsy in head injury cases: SAH> SDH> EDH EDH is the least found
in head injury. But EDH is exclusive for head injury- it generates upon heavy and
intense blow To head only. Trivial head injuries do not lead to EDH. 95% of EDH
are associated with skull fracture

Question: 176
All of the following are increased in iron deficiency anemia except?

https://emedicoz.com/testresult/683510s14692528 138/160
6/9/24, 12:52 PM Neet PG Preparation, Neet PG Coaching, FMGE, USMLE

A Transferrin saturation

B Urinary protoporphyrins

C Total iron binding capacity

D Serum soluble transferrin


receptors

46.34% People got this right

Explanation:

Correct Answer (A) Transferrin saturation. In IDA serum ferritin, serum iron and
transferrin saturation are reduced while options B, C D are increased.

Question: 177
A 38-year woman, mother of two children who has undergone
laparoscopic tubectomy 4 years back, presents with pain in the scar area.
Further history suggests that the pain coincides with the days of
menstruation. Physical examination reveals a swelling of 2x2 cm in the
scar area which is extremely tender and erythematous. A FNAC was
performed. Which of the following is most likely finding on FNAC?

A Suture material with


granulomas

B Benign glands with brown


pigment

C Glands with nuclear


pleomorphism

https://emedicoz.com/testresult/683510s14692528 139/160
6/9/24, 12:52 PM Neet PG Preparation, Neet PG Coaching, FMGE, USMLE

D Hook lets with


inflammation

37.93% People got this right

Explanation:

Correct Answer (B) Benign glands with brown pigment. The patient is presenting
with a classical scar endometriosis which has benign looking (endometrial)
glands and hemosiderin ( brown pigment). Option A is for suture granuloma,
pain will not be associated with menstruation, option C is for malignancy option
D is for cysticercosis.

Question: 178
A 10-year-boy presents with passing of red urine and pedal edema, BP is
140/90. Urine analysis shows RBCs and RBC casts in urine. Further the
child had multiple abscesses on skin 15 days back which have healed by
now. Which of the following is true about this disease? i. There is an
increased renal Na and water reabsorption in this child ii. There is an
increased synthesis of hepatic lipoproteins in this child. iii. Size based
filtration defect in the renal glomeruli iv. There is an increased GFR in this
child v. Acute renal failure is a rare complication of this disease.

A ii,iii,iv are true

B ii,iii,v are true

C i,iii,iv are true

D i,iii,v are true

19.83% People got this right

Explanation:

Correct Answer (D) i,iii,v are true. The child is having a post streptococcal
glomerulonephritis which manifests as nephritic syndrome which will have
https://emedicoz.com/testresult/683510s14692528 140/160
6/9/24, 12:52 PM Neet PG Preparation, Neet PG Coaching, FMGE, USMLE

increased renin leading to option i. option ii, is seen in Nephrotic syndromes. In


Nephritic syndrome GFr is reduced not increased

Question: 179
Hereditary non polyposis colon cancer is caused by:

A Inactivation of DNA
mismatch repair gene

B Mutation of SKT-11 tumor


suppressor gene

C Mutation of P53 gene

D Mutation of APC gene

41.18% People got this right

Explanation:

Correct Answer (A) Inactivation of DNA mismatch repair gene. • It is an


autosomal dominant condition characterized by the increased incidence of colon
cancer and extraintestinal cancer particularly the ovarian and endometrial
cancer. • Mutation in DNA repair genes (MSH2 and MLHL) leading to
microsatellite instability • Colon cancers in these patients affect right ot
ascending colon and.occurs at younger age (<50 years) The proximal colon
tumors in HNPCC have a better prognosis than sporadic tumors from patients of
similar age

Question: 180
A 2-year child presents with excessive urination, excessive thirst despite
drinking large amounts of fluid the child clinically appears dehydrated. X
ray shows lytic lesions in skull. Examination shows it is a neoplastic
condition. Which of the following is most likely diagnosis?

https://emedicoz.com/testresult/683510s14692528 141/160
6/9/24, 12:52 PM Neet PG Preparation, Neet PG Coaching, FMGE, USMLE

A Metastatic neuroblastoma

B Langerhan cell histiocytosis

C Acute lymphoblastic
leukaemia

D Plasma cell neoplasm

37.3% People got this right

Explanation:

Correct Answer (B) Langerhan cell histiocytosis

Question: 181
A 35-year smoker presents with dry cough, investigations did not reveal any
mass in the lungs. A biopsy from upper respiratory tract was done and is as
follows. Which of the following is not true about the phenomenon
happening?

A Happens only in epithelium

B Reversible

https://emedicoz.com/testresult/683510s14692528 142/160
6/9/24, 12:52 PM Neet PG Preparation, Neet PG Coaching, FMGE, USMLE

C Premalignant

D Happens after recurrent


injury

24.06% People got this right

Explanation:

Correct Answer (A) Happens only in epithelium. The history and mage are
suggestive of squamous metaplasia of respiratory tract. Metaplasia can occur
even in mesenchyme ( eg myositis ossificans)

Question: 182
Gross examination of liver in a patient with Cor pulmonale is as follows. True
about the finding in liver is?

A Red areas are hyperaemic

B Red areas are necrotic

C Pale areas are ischemic

D Pale areas are necrotic

https://emedicoz.com/testresult/683510s14692528 143/160
6/9/24, 12:52 PM Neet PG Preparation, Neet PG Coaching, FMGE, USMLE

26.49% People got this right

Explanation:

Correct Answer (B) Red areas are necrotic. This is a chronic venous congestion of
liver (not hyperaemia) which leads to haemorrhagic necrosis (red infarct) leading
to Nut meg appearance.

Question: 183
A clinical study is performed of oncogenesis in human neoplasms. It is
observed that some neoplasms appear to develop from viral oncogenesis,
with serologic confirmation of past viral infection of HTLV. Which of the
following neoplasms is most likely to arise in this manner?

A B- cell leukaemia

B Burkitt lymphoma

C AAngiocentric lymphoma

D T cell leukaemia

25.23% People got this right

Explanation:

Correct Answer (D) T cell leukaemia. Adult T cell leukaemia / lymphoma is


caused by HTLV option B and C are caused by EBV

Question: 184
True regarding vasculitis is/are I. HBV is etiology for polyarteritis nodosa
II. Henoch Schnolein Purpura is immune complex mediated III. Takayasu
arteritis presents with hand claudication IV. Microscopic polyangitis has
pulmonary vessel involvement V. Kawasaki vasculitis frequently occurs in
young children

https://emedicoz.com/testresult/683510s14692528 144/160
6/9/24, 12:52 PM Neet PG Preparation, Neet PG Coaching, FMGE, USMLE

A I,III,IV

B I,III,IV,V

C I,II,IV,IV

D All are true

40.14% People got this right

Explanation:

Correct Answer (D) All are true

Question: 185
Which of the following is the etiology for the gross finding seen in this
image?

A Acute glomerulonephritis

B Chronic glomerulonephritis

C Malignant hypertension

https://emedicoz.com/testresult/683510s14692528 145/160
6/9/24, 12:52 PM Neet PG Preparation, Neet PG Coaching, FMGE, USMLE

D Pyonephrosis

41.64% People got this right

Explanation:

Correct Answer (C) Malignant Hypertension. The image shows multiple petechial
images on the surface of the kidney which is knows as flea bitten kidney. Causes
include malignant hypertension , infective endocarditis, polyarteritis nodosa,
small vessel vasculitis (HSP), HUS-TTP and even RPGN.

Question: 186
Lepidic pattern of spread is seen in?

A Bronchiolo-alveolar
carcinoma

B Lobular carcinoma

C Adenoid cystic carcinoma

D Renal cell carcinoma

24.63% People got this right

Explanation:

Correct Answer (A) Bronchioloalveolar carcinoma. Lepidic pattern refers to


extension along the alveolar lining without invasion into the stroma seen in
adenocarcinoma insitu ( formerly known as bronchioloalveolar carcinoma.

Question: 187
Antibodies made in the spleen that are directed against the cell surface
antigens GpIIb/IIIa or GpIb/IX are characteristically seen in individuals

https://emedicoz.com/testresult/683510s14692528 146/160
6/9/24, 12:52 PM Neet PG Preparation, Neet PG Coaching, FMGE, USMLE

with?

A Bernard Soullier Syndrome

B Glansmann
thrombasthenia

C Both A & B

D ITP

13.74% People got this right

Explanation:

Correct Answer (D) ITP. In BSS there is a congenital deficiency of GpIb/IX and in
GT there is a congenital deficiency of GpIIb/IIIa.

Question: 188
Biopsy from a gastric tumor is as follows. Which of the following is likely
presentation of this patient?

A An ulceroproliferative mass
into the lumen of stomach

https://emedicoz.com/testresult/683510s14692528 147/160
6/9/24, 12:52 PM Neet PG Preparation, Neet PG Coaching, FMGE, USMLE

B A submucosal lesion with


no involvement of mucosa

C Loss of mucosal rugae


without obvious mass

D A patient with generalised


lymphadenopathy with stomach
involvement

17.67% People got this right

Explanation:

Correct Answer (C) Loss of mucosal rugae without obvious mass. Histology
shows signet ring cells which frequently present as linnitis plastica which is
typical description of option C.

Question: 189
Which of the following disease occurs due to antibodies to C3
convertase?

A Hereditary Angioneurotic
edema

B Dense deposit disease

C Wiskott Aldric Syndrome

D Systemic lupus
erythomatosis

21.74% People got this right

Explanation:

https://emedicoz.com/testresult/683510s14692528 148/160
6/9/24, 12:52 PM Neet PG Preparation, Neet PG Coaching, FMGE, USMLE

Correct Answer (B) Dense deposit Disease. Dense deposit disease or MPGN type
II occurs due to C3 Nephritic factor (C3NeF) which is an antibody to C3
convertase which reduces the lysis of C3 convertase leading to prolonged
activation of C3

Question: 190
A 38-year woman, mother of two children who has undergone laparoscopic
tubectomy 4 years back, presents with pain in the scar area. Further history
suggests that the pain coincides with the days of menstruation. Physical
examination reveals a swelling of 2x2 cm in the scar area which is extremely
tender and erythematous. A local excision of the same was done. Cut
section of the specimen showed a dark brown colored fluid pigment and a
special stain for the same is?

A Lipofuscin: Sudan Balck

B Melanin: Masson Fontana

C Collagen: Masson
Trichrome

D Hemosiderin: Prussian Blue

33.29% People got this right

Explanation:
https://emedicoz.com/testresult/683510s14692528 149/160
6/9/24, 12:52 PM Neet PG Preparation, Neet PG Coaching, FMGE, USMLE

Correct Answer (D) Hemosiderin: Prussian Blue. The history suggests it is a scar
endometriosis, histological image shows glands and hemorrhage confirming the
same. The special stain for hemosiderin (arising from hemorrhage) is Prussian
blue.

Question: 191
The stridor from vocal fold dysfunction is typically which of the
following?

A High-pitched, inspiratory,
or biphasic

B High-pitched, inspiratory
only

C Low-pitched, inspiratory, or
biphasic

D Low-pitched, inspiratory,
biphasic, or expiratory

19.77% People got this right

Explanation:

Correct Answer (A) High-pitched, inspiratory, or biphasic Ref: Read the text
below. Sol:- VOCAL FOLD DYSFUNCTION • The stridor of VFP is inspiratory or
biphasic, with a high-pitched musical quality. • Unilateral VFP typically presents
with a weak cry or stridor, aspiration, dysphagia, or feeding difficulties. Infants
with bilateral VFP often have severe airway obstruction that requires a
tracheotomy. • Approximately 70% of non iatrogenic unilateral VFP will resolve
spontaneously, most within the first 6 months of life.

Question: 192
Choose the most appropriate statement regarding epistaxis:

https://emedicoz.com/testresult/683510s14692528 150/160
6/9/24, 12:52 PM Neet PG Preparation, Neet PG Coaching, FMGE, USMLE

A Little's area is the anterior


end of the inferior turbinate.

B Kiesselbach's plexus is in
the pterygopalatine fossa.

C Bilateral epistaxis implies


bleeding from the nasopharynx.

D Bleeding above the level of


the middle turbinate is likely to
originate from the internal carotid
artery.

15.4% People got this right

Explanation:

Correct Answer (D) Bleeding above the level of the middle turbinate is likely to
originate from the internal carotid artery. Ref: Read the text below. Sol:- • Little's
area is the anterior part of the septum. • Kiesselbach's plexus is in Little's area. •
Bilateral epistaxis is commonly due to the original side being partially blocked by
the patient or by clot; blood then passes behind the choana and down the
opposite nasal fossa. Alternatively there may be more than one site, but
bleeding from the nasopharynx is rare. • From the anterior ethmoidal, a branch
of the ophthalmic, supplied from the internal carotid.

Question: 193

https://emedicoz.com/testresult/683510s14692528 151/160
6/9/24, 12:52 PM Neet PG Preparation, Neet PG Coaching, FMGE, USMLE

Structure shown in image is attached to

A Scala Tympani

B Scala Vestibuli

C Incus

D Malleus

20.4% People got this right

Explanation:

Correct Answer (C)

Incus

Ref: Read the text below

Sol:

IMPLANTABLE HEARING AIDS

Vibrant sound bridge device

The external component = audio processor


The internal component is called vibrating ossicular prosthesis (VORP) and =the
receiver, floating mass transducer (FMT) and a conductor link between the two.
FMT --- INCUS

https://emedicoz.com/testresult/683510s14692528 152/160
6/9/24, 12:52 PM Neet PG Preparation, Neet PG Coaching, FMGE, USMLE

Question: 194
A 46-year-old woman reports with a 9-year history of progressive hearing
loss in the right ear. An audiogram shows mild to moderate sensorineural
hearing loss in the right ear, and an MRI of the brain shows an enhancing
mass on the right that erodes the posterior face of the petrous bone.
What other diagnostic studies would be appropriate in this patient?

A Echocardiogram

B Genetic testing to evaluate


for mutation on chromosome 22

C Renal ultrasound

D Vestibular-evoked
myogenic potential testing

7.18% People got this right

Explanation:

Correct Answer (C) Renal ultrasound Ref: Read the text below. Sol:- • Aggressive
papillary adenocarcinoma of the endolymphatic sac may erode the posterior

https://emedicoz.com/testresult/683510s14692528 153/160
6/9/24, 12:52 PM Neet PG Preparation, Neet PG Coaching, FMGE, USMLE

face of the petrous bone and may be associated with von Hippel–Lindau disease
and renal cyst or tumors.

Question: 195
Which is the most common type of congenital ossicular dysfunction?

A Isolated stapes defect

B Stapes defect with fixation


of footplate and lenticular process
involvement

C Defective lenticular process


of incus

D None of the above

36.13% People got this right

Explanation:

Correct Answer (B) Stapes defect with fixation of footplate and lenticular process
involvement Ref– Read the text below Sol: • The most common congenital
isolated ossicular anomalies are stapes fixation and incudostapedial
discontinuity , and isolated congenital stapes fixation represents 20% to 35% of
ossicular malformations. • It is of a great interest to note that isolated stapes
footplate fixation comprised more than 50% in this study. • Congenital stapes
fixation is the result of a fixation between the peripheral lamina stapedialis and
the annular ligament, and it is the cause of conductive deafness with a normal
tympanic membrane. • The differential diagnosis of stapes fixation includes
other ossicular malformations, oval or round window atresia, congenital
cholesteatoma at early stage or other middle ear tumors, and ossicular trauma. •
Congenital stapes fixation must be distinguished from stapes otosclerosis which
is defined by a progressive deafness, even if the surgical procedures are the
same.

https://emedicoz.com/testresult/683510s14692528 154/160
6/9/24, 12:52 PM Neet PG Preparation, Neet PG Coaching, FMGE, USMLE

Question: 196
A 40-year-old man presents with a history of unilateral offensive ear
discharge for 6 months with squamous debris in the epitympanum that is
not self-cleaning. A perforation is seen in the pars flaccida. What is the
most likely diagnosis?

A Squamous cell carcinoma

B Chronic otitis media


Atticoantral type

C Chronic otitis media


tubotympanic type

D Malignant otitis externa

50.33% People got this right

Explanation:

Correct Answer (B)

Chronic otitis media Atticoantral type

Ref– Read the text below

Sol:

Squamous debris in the epitympanum that is not self-cleaning and a perforation


seen in the pars flaccida go in favour of atticoantral disease.

https://emedicoz.com/testresult/683510s14692528 155/160
6/9/24, 12:52 PM Neet PG Preparation, Neet PG Coaching, FMGE, USMLE

Question: 197
Hennebert's sign is a false positive fistula test when there is no evidence
of middle ear disease causing fistula of horizontal semicircular canal, it is
seen on?

A Congenital syphilis

B Stapedectomy

C Meniere's disease

D Cholesteatoma

33.15% People got this right

Explanation:

Correct Answer (A)

Congenital syphilis

Ref– Read the text below

Sol:

https://emedicoz.com/testresult/683510s14692528 156/160
6/9/24, 12:52 PM Neet PG Preparation, Neet PG Coaching, FMGE, USMLE

Question: 198
Resonance Frequencies are natural frequencies that vibrate a mass with
the least amount of force. Resonance frequency for Concha is

A 800–1600 Hz

B 800 Hz.

C 4000–6000 Hz

D 500–2000 Hz

15.54% People got this right

Explanation:

Correct Answer (C) 4000–6000 Hz Sol: Resonance Frequencies: natural


frequencies that vibrate a mass with the least amount of force • Concha: 4000–
6000 Hz • EAC: 2000–3000 Hz • TM: 800–1600 Hz • Ossicles: 500–2000 Hz •
Middle Ear: 800 Hz

Question: 199
Penetration of which structure allows infection to spread from the
paranasal sinuses to the orbit?

https://emedicoz.com/testresult/683510s14692528 157/160
6/9/24, 12:52 PM Neet PG Preparation, Neet PG Coaching, FMGE, USMLE

A Basal lamella

B Fovea ethmoidalis

C Uncinate process

D Lamina papyracea

41.13% People got this right

Explanation:

Correct Answer (D)

Lamina papyracea

Reference – Otolaryngology by R.Pasha --5

Sol:

Lamina Papyracea:

Lateral thin bony wall of the ethmoid sinus, separates orbit from ethmoid cells as
a part of the medial orbital wall
Computerized axial tomography (CAT) radiograph of the nose.Coronal section
view of the osteomeatal unit.

Fovea Ethmoidalis:

Roof of ethmoid sinus


Ground (Basal) Lamella:

Posterior bony insertion of the middleturbinate which separates anterior and


posterior ethmoid cells; posterior extension of the middle turbinate
Uncinate Process:

Sickle-shaped thin bone part of the ethmoid bone, covered by mucoperiosteum,


medial to the ethmoidinfundibulum and lateral to the middle turbinate(derived
from the second ethmoidal turbinal)

https://emedicoz.com/testresult/683510s14692528 158/160
6/9/24, 12:52 PM Neet PG Preparation, Neet PG Coaching, FMGE, USMLE

Question: 200
Abscess shown in the Fig spares :

A CN IX

B CN X

C CN XI

D CN VII

39.16% People got this right

Explanation:

Correct Answer (D) CN VII Sol: • The parapharyngeal space may be divided into
two compartments on the basis of its relationship to the styloid process or, more

https://emedicoz.com/testresult/683510s14692528 159/160
6/9/24, 12:52 PM Neet PG Preparation, Neet PG Coaching, FMGE, USMLE

precisely, to the tensor-vascular-styloid fascia. • The importance of the


parapharyngeal space also lies in its relationship with the other spaces of the
neck. The masticator and parotid spaces are located laterally, the pharyngeal
mucosal space is located medially, and the retropharyngeal space is located
posteromedially. • The contents of the prestyloid compartment include the
minor or ectopic salivary gland, branches of the mandibular division of the
trigeminal nerve, internal maxillary artery, ascending pharyngeal artery, and
pharyngeal venous plexus, whereas those of the poststyloid compartment
include the internal carotid artery, internal jugular vein, cranial nerves IX-XII,
cervical sympathetic chain, and glomus bodies. • The facial nerve (CN VII) has
only a brief course through the poststyloid space as it exits the skull via the
stylomastoid foramen

https://emedicoz.com/testresult/683510s14692528 160/160

You might also like